Microbiology step 1

Pataasin ang iyong marka sa homework at exams ngayon gamit ang Quizwiz!

A 17-year-old girl comes to the physician because of a 1-week history of severe itching in the area of her genitals. She reports that the itching is most severe at night. She has been sexually active with three partners over the past year; she uses condoms for contraception. Her current sexual partner is experiencing similar symptoms. Pelvic examination shows vulvar excoriations. A photomicrograph of an epilated pubic hair is shown. Which of the following is the most likely causal organism? a)Phthirus pubis b)Sarcoptes scabiei c)Enterobius vermicularis d)pédiculés humanus

( -Infestation with the parasite Phthirus pubis causes pediculosis pubis, an infection that manifests with anogenital pruritus. Pediculosis pubis is a sexually transmitted infection that is not prevented by condoms. Physical examination of the pubic area typically shows excoriations as well as lice, their nits, and their droppings, which are visible both with the naked eye and on microscopy, as seen in the photomicrograph here.

A 35-year-old woman that is currently being treated for chronic hepatitis C comes to the physician because of progressive fatigue for 10 days. Examination shows pale conjunctivae. Her hemoglobin concentration is 10.1 g/dL, serum total bilirubin concentration is 1.9 mg/dL, and LDH is 259 U/L (N = 45-90 U/L). Which of the following mechanisms most likely contributes to the efficacy of this patient's pharmacotherapy? a)Activation of leukocytes b)Decreased release of progeny virus c)Inhibition of reverse transcriptase d)Inhibition of RNA polymerase e)Decreased GMP synthesis

(A combination of decreased hemoglobin, elevated bilirubin, and elevated LDH indicates hemolytic anemia, which is a known side effect of ribavirin.) - Decreased GMP synthesis: Ribavirin is a nucleoside analog with multiple effects on HCV. One of the most well-studied is competitive inhibition of IMP dehydrogenase, the enzyme that produces GMP. Because GMP is used to create viral RNA, decreased GMP synthesis leads to impaired viral replication of HCV. Another important mechanism of action is the insertion of ribavirin into the HCV viral genome, leading to increased mutation rates.

A 1-year-old girl is brought to the physician for a well-child examination. She has no history of serious illness. She receives a vaccine in which a polysaccharide is conjugated to a carrier protein. Which of the following pathogens is the most likely target of this vaccine? a)Corynebacterium diphtheriae b)Clostridium tetani c)Hepatitis A virus d)Varicella zoster virus e)Streptococcus pneumoniae f)Bordetella pertussis

(A conjugate vaccine contains capsular polysaccharide antigens that are conjugated to carrier proteins, which enhances immunogenicity by promoting T-cell activation.) - Streptococcus pneumoniae: Conjugate polysaccharide vaccines are used to target encapsulated bacteria, such as Streptococcus pneumoniae. In young children (< 2 years), capsular polysaccharide antigens do not produce a sufficient immune response. Therefore, conjugate vaccines are administered to ensure sufficient immunization via activation of T cells, which stimulate a more rapid and long-lasting immune response. In addition to the PCV13, other conjugate vaccines include the Hib vaccine and meningococcal vaccine.

A 66-year-old woman comes to the emergency department because of fever and difficulty swallowing for 5 hours.She appears anxious. Her temperature is 39.1°C (102.4°F). Physical examination shows an extended neck and excessive drooling. Her voice is muffled and there is inspiratory stridor. There is tender bilateral cervical lymphadenopathy and pain upon palpation of the hyoid. Laboratory studies show a leukocyte count of 18,800/mm3with 85% neutrophils. Which of the following is the most likely causal organism? a) haemophilus influenzae type b b)strep pyogenes c)adenovirus d)corynebacterium diphteriae e)parainfluenza

(A lateral cervical x-ray of patient's with this condition often shows an enlarged epiglottis and supraglottic narrowing, also known as a "thumbprint sign". Various diseases present with inspiratory stridor as a main diagnostic feature, but only epiglottitis is notable for painful swallowing, head extension, excessive drooling, a muffled voice, and the absence of a cough.) - haemophilus influenzae type b: Fever, painful swallowing, excessive drooling, a muffled voice, and tender cervical lymphadenopathy are suggestive of epiglottitis. Although this condition is more common in children between 6 and 12 years of age, it can occur at any age. Haemophilus influenzae type b (Hib) used to be the most common cause of epiglottitis in the US, but since the introduction of the Hib vaccine, other bacteria (S. pyogenes, S. pneumoniae, S. aureus) are also common in pediatric cases. However, Hib is still the most common cause of epiglottitis in adults (25% of cases), the majority of which have not yet received vaccination.

A 35-year-old man comes to the physician because of an ulcer on his penis that he first noticed 4 days ago. He is currently sexually active with multiple male partners and uses condoms inconsistently. Genital examination shows a shallow, nontender ulcer with a smooth base and indurated border along the shaft of the penis. There is bilateral inguinal lymphadenopathy. Darkfield microscopy of a sample from the lesion shows gram-negative, spiral-shapedbacteria. A drug that acts by inhibition of which of the following is the most appropriate treatment for this patient? a)Transpeptidase b)Aminoacyl-tRNA binding c)Dihydrofolate reductase d)Dihydropteroate synthase e)Nucleic acid synthesis

(A nontender genital ulcer (chancre) with associated inguinal lymphadenopathy suggests a diagnosis of primary syphilis. The presence of spirochetes on darkfield microscopy confirms infection with Treponema pallidum, the causal pathogen of syphilis. The first-line therapy for this condition is intramuscular benzathine penicillin G.) - Penicillin G, a beta-lactam antibiotic, binds to penicillin-binding proteins (transpeptidase) and inhibits cross-linking of peptidoglycan during bacterial cell wall synthesis. It has antimicrobial activity against streptococci, pneumococci, meningococci, and Treponema pallidum.

A previously healthy 25-year-old man comes to the physician because of a 1-week history of fever and fluid release from painful lumps in the right groin. He had an atraumatic ulceration of the penis about 1 month ago that was not painful and resolved on its own within 1 week. He works at an animal shelter for abandoned pets. He is sexually active with multiple male partners and does not use condoms. His temperature is 38.5°C (101.3°F). Examination of the groin shows numerous, tender nodules with purulent discharge. The remainder of the examination shows no abnormalities. Which of the following is the most likely causal pathogen? a) haemophilus ducreyi b)chlamydia trachoma's c)treponema d)bartonella henselae

(A painless penile lesion followed by suppurative lymphadenopathy 2-4 weeks after the initial lesion has healed is consistent with a diagnosis of lymphogranuloma venereum.) -Chlamydia trachomatis: The serotypes L1, L2, and L3 of Chlamydia trachomatis cause lymphogranuloma venereum, the incidenceof which is increased in men who have sex with men. Further manifestations include systemic symptoms such as fever, malaise, chills, and/or myalgia. Lymphogranuloma venereum is treated with doxycycline or macrolides such as erythromycin. Without antibiotic treatment, lymphogranuloma venereum can cause chronic genital lymphedema, strictures in the anogenital tract, and/or infertility.

A 56-year-old man is brought to the emergency department because of a 4-day history of fever, chills, headache, and myalgia. This morning, he also noticed a painful lesion on his thumb. Last week, he went on a trip to Missouri where he was hunting rabbits. His temperature is 39.4°C (102.9°F), pulse is 108/min, and blood pressure is 137/83 mm Hg. Physical examination shows a 2-cm ulcer with a central eschar on his right thumb surrounded by an erythematous circular rim. There is pronounced tender right trochlear and axillary lymphadenopathy. Laboratory studies show a four-fold increase in F. tularensis-specific antibody titers. The most appropriate antibiotic for this patient's condition acts on which of the following processes? a)Synthesis of RNA b)Formation of cell wall peptidoglycan c)Assembly of ribosomal subunit d)Transpeptidase cross-linking in cell wall

(A patient with fever, malaise, an ulcer, and evidence of F. tularensis on laboratory studies most likely has tularemia, which can be treated with aminoglycosides) -The assembly of ribosomal 30s subunit is inhibited by aminoglycosides (e.g., gentamicin and streptomycin), which are the drugs of choice for tularemia. Depending on the specific aminoglycoside, formation of the initiation complex or the translocation process is inhibited, resulting in the inhibited assembly of the ribosomal subunit and impaired bacterial protein synthesis. This patient was most likely infected by a tick, e.g., the American dog tick or the Lone star tick, or touched an infected rabbit during his hunting trip. Typically, this pathogen causes an ulcer at its entry point, tender regional lymphadenopathy, and high fever, all of which are seen in this patient.

An otherwise healthy 25-year-old woman comes to the physician because of a 2-day history of pain and swelling of her right knee joint and left wrist. She went camping with her new boyfriend 3 weeks ago but does not recall any tick bites. Her temperature is 37.8°C (100.0°F). Examination of the right knee shows swelling, warmth, and tenderness on passive movement. There is a tender pustule on the sole of the left foot. Arthrocentesis of the right knee joint yields 8 mL of cloudy fluid with a leukocyte count of 45,000/mm3 (90% segmented neutrophils with intracellular organisms). Which of the following is the strongest risk factor for this patient's condition? a)Autoantibody production b)Intravenous drug use c)Sexually transmitted infection d)Ixodes tick bite

(A positive nucleic acid amplification test (NAAT) would confirm the diagnosis in this case.) - sexually transmitted: Gonorrhea is a sexually transmitted infection caused by Neisseria gonorrhoeae, which can either manifest in a genitourinary, often asymptomatic form or as disseminated disease with arthritis-dermatitis syndrome or purulent arthritis. Arthritis-dermatitis syndrome typically presents with migratory, asymmetric arthritis, tenosynovitis, and vesicular, pustular, or maculopapular lesions on the trunk and extremities (sometimes involving the palms and soles), as seen in this patient. Synovial fluid analysisshowing an increased neutrophil-dominated leukocyte count with intracellular organisms further supports the diagnosis.

A previously healthy 53-year-old man is brought to the emergency department 45 minutes after the onset of a severe headache. He returned from a vacation in the mountains 4 days ago, during which he went swimming in a freshwater lake. On arrival, he is confused. His temperature is 39°C (102.2°F) and blood pressure is 105/68 mm Hg. Neurologic examination shows diffuse hyperreflexia. An MRI of the brain shows asymmetrical, bitemporal hyperintensities. A lumbar puncture is performed. Cerebrospinal fluid analysis shows leukocyte 120 segmented neutrophils 10% lymphocytes 90% glucose 45 opening pressure 130 protein 75 most likely causal pathogen? a)Naegleria fowleri b)Tick-borne encephalitis virus c)Neisseria meningitidis d)West Nile virus e)Cytomegalovirus f)Herpes simplex virus g)La Crosse virus h)Rabies virus

(Acute illness associated with headache, fever, altered mental status, and CSF showing lymphocytic pleocytosis with normal glucose levels and elevated protein levels is indicative of acute viral encephalitis. The localized temporal abnormalities seen on MRI are virtually pathognomonic of this pathogen) - Herpes Simplex Virus: Herpes simplex virus is the most common cause of viral encephalitis. MRI of the brain classically shows temporal lobe involvement, possibly including hemorrhagic changes. Damage to the temporal lobe is also responsible for some of the other clinical features that may be seen in herpes simplex encephalitissuch as olfactory hallucinations, aphasia, and focal seizures.

A previously healthy 53-year-old man is brought to the emergency department 45 minutes after the onset of a severe headache. He returned from a vacation in the mountains 4 days ago, during which he went swimming in a freshwater lake. On arrival, he is confused. His temperature is 39°C (102.2°F) and blood pressure is 105/68 mm Hg. Neurologic examination shows diffuse hyperreflexia. An MRI of the brain shows asymmetrical, bitemporal hyperintensities. A lumbar puncture is performed. Cerebrospinal fluid analysis shows: leukocyte 120 neutrophils 10% lymphocytes 90% glucose 45 op 130 protein 75 Which of the following is the most likely causal pathogen? a)Naegleria fowleri b)Tick-borne encephalitis virus c)Neisseria meningitidis d)West Nile virus e)Cytomegalovirus f)Herpes simplex virus

(Acute illness associated with headache, fever, altered mental status, and CSF showing lymphocytic pleocytosis with normal glucose levels and elevated protein levels is indicative of acute viral encephalitis. The localized temporal abnormalities seen on MRI are virtually pathognomonic of this pathogen.) - Herpes simplex virus is the most common cause of viral encephalitis. MRI of the brain classically shows temporal lobe involvement, possibly including hemorrhagic changes. Damage to the temporal lobe is also responsible for some of the other clinical features that may be seen in herpes simplex encephalitissuch as olfactory hallucinations, aphasia, and focal seizures.

A 26-year-old man comes to the physician because of fever and right leg pain and swelling for the past 3 days. Two weeks ago, he had a short episode of abdominal pain and diarrhea that resolved without treatment. He is an avid hunter and went hunting for wild boar with his father 3 weeks ago. He took some of the meat home for consumption. His temperature is 38.6°C (101.5°F). Examination shows periorbital edema and conjunctival hemorrhage. Palpation of the right calf elicits tenderness. Range of motion of the right ankle is restricted by pain. A sample of which of the following is most likely to show the causal pathogen? a)muscle b)stool c)blood d)lymph node e)skin

(An episode of abdominal pain and diarrhea followed by fever, leg pain and swelling, periorbital edema and conjunctivalhemorrhage in a patient who consumed wild boar meat suggests trichinellosis) - muscle: Trichinellosis is a helminth infection most commonly caused by ingesting the encysted larvae of Trichinella spiralis from undercooked meat (especially pork). The larvae invade the small bowel mucosa, where they develop into adult roundworms and may cause intestinal symptoms. New larvae released by adult worms then migrate to striated muscles, where they encyst and cause myositis, often leading to severe muscle pain, as seen here. T. spiralis larvae are most likely to be seen in a sample of muscle tissuefrom the involved portion of this patient's right calf.

A 28-year-old man comes to the physician for a pre-employment examination. He has no history of serious illness and takes no medications. A screening blood test is performed in which peptides are added to the sample to stimulate in vitro production of interferon-gamma, which is then measured using an enzyme-linked immunosorbent assay. This test is most likely to be helpful in diagnosing infection with which of the following pathogens? a)Human immunodeficiency virus b)Hepatitis B virus c)Mycobacterium tuberculosis d)poxvirus

(An interferon-γ release assay (IGRA) was the test performed here.) - The IGRA test for latent Mycobacterium tuberculosis relies on the fact that IFN-γ is released by T lymphocytes when they are exposed to mycobacterial antigens. IGRA is preferred in patients who have received Bacillus Calmette-Guérin (BCG) vaccine, as the test is not dependent on the patient's vaccination status (the PPD test may show a false positive in these patients). If an IGRA test is positive, further evaluation for active TB (e.g. chest x-ray) is indicated.

A 4-year-old girl is brought to the physician with progressively worsening fever, malaise, and a sore throat. Her parents say "Our daughter has not received vaccinations because her body has to learn how to fight infections." Her temperature is 38.5˚C (101.3 F). Physical examination shows marked cervical lymphadenopathy. There are gray-white membranes over the tonsils and posterior pharynx that bleed when scraped off. The patient's symptoms are most likely caused by disruption of which of the following steps in protein synthesis? a)Folding of completed proteins b)Translocation of the ribosome along the mRNA c)Binding of tRNA to the A site d)Formation of the initiation complex

(An unvaccinated patient presenting with fever, malaise, severe lymphadenopathy, and sore throat with grayish-white pharyngealpseudomembranes is diagnostic of infection with Corynebacterium diphtheriae.) - Translocation of the ribosome along the mRNA: Ribosomal translocation occurs during the elongation phase of translation, wherein the ribosome moves one triplet at a time along the mRNA in the 3' direction. Diphtheria toxin is a bacterial AB toxin that inactivates the process of translocation by adding a molecule of ADP-ribose to elongation factor 2. EF-2is then unable to catalyze GTP hydrolysis and fails to provide the ribosome sufficient energy to translocate by one codon. Protein synthesis is subsequently arrested, leading to cell death and necrosis.

A 72-year-old man comes to the physician with chills, nausea, and diffuse muscle aches for 3 days. His niece had similar symptoms 2 weeks ago and H1N1 influenza strain was isolated from her respiratory secretions. He received his influenza vaccination 2 months ago. His temperature is 38°C (100.4°F). A rapid influenza test is positive. Which of the following mechanisms best explains this patient's infection despite vaccination? a)Random point mutations within viral genome b)Complementing with functional viral proteins c)Exchange of viral genes between chromosomes d)Reassortment of viral genome segments

(Antigenic drift is most likely the cause of the this patient's infection despite appropriate vaccination. It is typically the cause of influenza epidemics.) - Random point mutations within the viral genome are responsible for antigenic drift, which creates a new virus strain. In some cases, although the patient has been vaccinated against the previous strain, the new strain will have different antigenic properties that can evade the host immune response and cause an infection, as observed in this patient.

An investigator is studying bacterial adaptations to the environment in an infectious isolate and a noninfectious isolate of Hemophilus influenzae type B. Animals exposed to the noninfectious isolate do not develop symptoms of infection. The investigator cultivates the noninfectious isolate in a culture with lysed bacteria from the infectious isolate. Some of the animals exposed to these bacteria develop symptoms of infection. Which of the following processes is most likely responsible for the described findings? a)Transposition b)Specialized transduction c)Conjugation d)Reassortment e)Transformation

(Bacterial lysis releases naked DNA that can be taken up by other bacteria. Through this process, nonvirulent types of Hemophilusinfluenzae can obtain genes that encode the bacterial capsule, which allows the bacteria to become virulent.) - Bacterial transformation is a process in which bacteria take up naked DNA (e.g., from lysis of bacteria) through the cell membrane, incorporate the DNA into their genome, and express new genes. Several species of bacteria can carry out this process, including S. pneumoniae, Neisseria spp., and H. influenzaetype b, as seen in this experiment.

A 4-year-old boy is brought to the physician because of a 3-day history of fever and left ear pain. Examination of the left ear shows a bulging tympanic membrane with green discharge. Gram stain of the discharge shows a gram-negative coccobacillus. The isolated organism grows on chocolate agar. The causal pathogen most likely produces a virulence factor that acts by which of the following mechanisms? a)Binding of the Fc region of immunoglobulins b)Inactivation of 60S ribosome c)Overactivation of adenylate cyclase d)Cleavage of secretory immunoglobulins e)Inactivation of elongation factor

(Chocolate agar is a growth medium enriched with hemin (factor X) and nicotinamide adenine dinucleotide (factor V). It is used to isolate fastidious organisms like Haemophilus influenzae, a gram-negative coccobacillus that is a typical causative pathogen of otitis media.) - Cleavage of secretory immunoglobulins: Hemophilus influenza produces IgA protease, a virulence factor that mediates cleavage of secretory immunoglobulins (IgA antibodies). IgA protease thereby facilitates adherence of the pathogen to mucosal membranes (e.g., the middle ear mucosa and tympanic membrane), which are normally protected by IgA antibodies.

A 62-year-old woman with type 2 diabetes mellitus is brought to the emergency department by her husband because of fever, chills, and purulent drainage from a foot ulcer for 2 days. Her hemoglobin A1c was 15.4%16 weeks ago. Physical examination shows a 2-cm ulcer on the plantar surface of the left foot with foul-smelling, purulent drainage and surrounding erythema. Culture of the abscess fluid grows several bacteria species, including gram-negative, anaerobic, non-spore-forming bacilli that are resistant to bile and aminoglycoside antibiotics. Which of the following is the most likely source of this genus of bacteria? a)Stomach b)Skin c)Vagina d)Oropharynx e)Colon

(Culture of this patient's abscess fluid grows Bacteroides species.) - Colon: This patient most likely has a wound infection with Bacteroides fragilis, an anaerobic bacterium that is normally found in the colon. B. fragilis is a common pathogen in infected diabetic ulcers, particularly in the setting of a polymicrobial infection as seen in this patient. The additional infecting bacteria are necessary, as aerobic bacteria are required to create the tissue conditions necessary for an anaerobicinfection.

A 32-year-old man comes to the physician with difficulty swallowing for several weeks. Examination of the oropharynx shows lesions on palate and tongue that can be easily scraped off. An image of the lesions is shown. Which of the following is a risk factor for this patient's findings? a)Decline in CD4+ T-cells b)Inhalation of salbutamol c)Missed childhood vaccination d)Chronic nicotine abuse e)Epstein-Barr virus infection

(Curdy, white oral patches that can be easily scraped off suggest thrush) - Decline in CD4+ T-cells: Oral thrush is caused by Candida albicans infection. The risk of mucocutaneous candidiasis is increased in patients with T-cell deficiency, which can occur in conditions such as HIV infection (↓ CD4+ T-cellcount), severe combined immunodeficiency, and DiGeorge syndrome. Other risk factors for mucocutaneous candidiasis are uncontrolled diabetes mellitus, inhalation of corticosteroids, antibioticuse, and neonatal age group (Candida infection acquired during vaginal delivery). Mild oral thrush can be treated topically with nystatin ("swish and swallow"). The odynophagia seen in this patient could suggest involvement of the esophagus (candidal esophagitis). Severe thrush or candidal esophagitis will require systemic therapy with fluconazole or echinocandins.

Two months after giving birth to a boy, a 27-year-old woman comes to the physician with her infant for a well-childexamination. She was not seen by a physician during her pregnancy. Physical examination of the mother and the boy shows no abnormalities. Laboratory studies show elevated titers of hepatitis B surface antigen in both the mother and the boy. Which of the following statements regarding the infant's condition is most accurate? a)Hepatitis B e antigen titer is likely undetectable b)Chronic infection is unlikely c)Lifetime risk of hepatocellular carcinoma is low d)The viral replication rate is low e)Significant elevation of transaminases is not expected f)Breastfeeding should be avoided

(Elevated titers of HbsAg in both the mother and the infant suggest vertical transmission of the hepatitis B virus (HBV). - Transaminases are usually normal or only mildly elevated in cases of vertical transmission of HBV. Because HBV is not cytopathic itself and newborns lack mature cytotoxic T-cells that mediate damage infected hepatocytes, the degree of hepatic tissue damage will be very limited. Rarely, infected neonates may develop acute hepatitis B, which is usually mild and self-limited.

The occupational health department at a hospital implements new safety precautions to prevent laboratory-acquired infections. One of the new precautions includes disinfecting the microbiology laboratory benches with 70% ethanol before and after use. This measure is most likely to be effective in preventing the transmission of which of the following viruses? a)Hepatitis A virus b)Parvovirus c)Poliovirus d)Polyomavirus e)Herpes simplex virus

(Ethyl alcohol at concentrations of 60-80% is a potent virucidal agent that is effective against enveloped viruses) - Herpes simplex virus: Alcohols and other organic solvents disrupt plasma membranes and are effective in inactivating nearly all enveloped viruses, including herpesviruses (e.g., herpes simplex virus). The important exception is rhabdoviruses (e.g., rabies virus), which is resistant to ethyl alcohol but can be inactivated by 70% isopropyl alcohol. Ethyl alcohol at a concentration of 60-80% has also been shown to inactivate certain non-enveloped viruses such as adenovirus, rhinovirus, and rotavirus, but most other non-enveloped viruses are typically resistant to inactivation by alcohol.

A 7-year-old boy with a history of cystic fibrosis is brought to the physician for evaluation of recurrent episodes of productive cough, wheezing, and shortness of breath over the past month. Physical examination shows coarse crackles and expiratory wheezing over both lung fields. Serum studies show elevated levels of IgE and eosinophilia. A CT scan of the lungs shows centrally dilated bronchi with thickened walls and peripheral airspace consolidation. Antibiotic therapy is initiated. One week later, the patient continues to show deterioration in lung function. A sputum culture is most likely to grow which of the following? a)Dimorphic, broad-based budding yeast b)Monomorphic, septate hyphae that branch at acute angles c)Dimorphic, cigar-shaped budding yeast d)Monomorphic, narrow budding encapsulated yeast e)Monomorphic, broad, nonseptate hyphae that branch at wide angles f)Dimorphic, budding yeast with pseudohyphae

(Failure to respond to antibiotic treatment in a patient with a history of cystic fibrosis, recurrent pulmonary exacerbations, elevated IgE levels, and CT findings of bronchiectasis should raise concern for allergic bronchopulmonary aspergillosis (ABPA). - Monomorphic, septate hyphae that branch at acute angles are characteristic of Aspergillus fumigatus, the causative organism in allergic bronchopulmonary aspergillosis. ABPA most commonly affects patients with cystic fibrosis or severe asthma. Small and ubiquitous spores of Aspergillus easily reach the alveoli where they trigger a chronic and complex hypersensitivity reaction, primarily due to poor mucociliary clearance. Symptoms of ABPA include wheezing, chronic cough, and shortness of breath. Imaging most often shows evidence of central bronchiectasis, as seen in this patient. ABPA is treated with glucocorticoids and itraconazole.

A 59-year-old man comes to the physician because of urinary frequency and perineal pain for the past 3 days. During this time, he has also had pain with defecation. He is sexually active with his wife only. His temperature is 39.1°C (102.3°F). His penis and scrotum appear normal. Digital rectal examination shows a swollen, exquisitely tender prostate. His leukocyte count is 13,400/mm3. A urine culture obtained prior to initiating treatment is most likely to show which of the following? a) Gram-negative, lactose-fermenting rods in pink colonies b)Gram-negative, oxidase-positive rods in green colonies c) Gram-negative, encapsulated rods in mucoid colonies d)Gram-negative, aerobic, intracellular diplococci

(Fever, increased urinary frequency, and a swollen, tender prostate are consistent with acute bacterial prostatitis. Given this patient's age and lack of risk factors, the most likely infectious agent is Escherichia coli.) - Gram-negative, lactose-fermenting rods in pink colonies: E. coli is a gram-negative, lactose-fermenting rod that grows in pink colonies on MacConkey agar. E. coliis the most common etiology of acute bacterial prostatitis in men older than 35 years of age.

A 69-year-old woman comes to the emergency department because of a 2-day history of cough and dyspnea. The cough is productive of small amounts of green phlegm. She has stage IV colon cancer and chronic obstructive pulmonary disease. Her medications include 5-fluorouracil, leucovorin, a fluticasone-salmeterol inhaler, and a tiotropium bromide inhaler. Her temperature is 39°C (102.2°F), pulse is 107/min, respirations are 31/min, and blood pressure is 89/68 mm Hg. Pulse oximetry on room air shows an oxygen saturation of 87%. Pulmonary examination shows diffuse crackles and rhonchi. An x-ray of the chest shows a left upper-lobe infiltrate of the lung. Two sets of blood cultures are obtained. Endotracheal aspirate Gram stain shows gram-negative rods that are oxidase-positive. Two large bore cannulas are inserted and intravenous fluids are administered. Which of the following is the most appropriate pharmacotherapy? a)Ertapenem and gentamicin b)Ceftriaxone and azithromycin c)Cefepime and levofloxacin

(Fever, productive cough, crackles, and an infiltrate on chest x-ray are consistent with community-acquired pneumonia (CAP). Given this patient's history of COPD, possible immunosuppression due to chemotherapy, greenish sputum, and gram-negative, oxidase-positive rods, Pseudomonas aeruginosa is the most likely causative organism.) - Cefepime and levofloxacin are the first-line treatment for patients with CAP when there is a strong suspicion of P. aeruginosa infection. Other treatment options for CAP with P. aeruginosa include piperacillin/tazobactam, meropenem, or imipenem plus a fluoroquinolone. An aminoglycoside may be added as well.

Four days after admission to the hospital for pneumonia, a 68-year-old woman develops abdominal pain and watery, foul-smelling diarrhea. Current medications include intravenous ceftriaxone and oral azithromycin. Stool cultures grow gram-positive, anaerobic rods. She is moved to an isolated hospital room. Sterilization with which of the following agents is most likely to prevent transmission of this pathogen to the next patient who will occupy her room? a)Chlorine-based solution b)Chlorhexidine-based solution c)Iodine-based solution

(Foul-smelling, watery diarrhea after antibiotic treatment is consistent with Clostridium difficile colitis. The appropriate disinfection method is both bactericidal and sporicidal.) - Chlorine-based solutions (such as bleach) are both bactericidal and sporicidal and can be used to sterilize surfaces and equipment that has been potentially contaminated by Clostridioides difficile. C. difficile spores can remain on dry surfaces for several months and are unaffected by typical sanitary products (e.g., alcohol-based solutions) or normal boiling. The protocol to minimize risk of infection and spread of C. difficile within hospital settings involves isolation of the patient, implementation of contact-precautions, and proper handwashing with soap and water before and after each encounter. Autoclaving is also sporicidal and can be used to sterilize larger equipment that has come in contact with this pathogen.

An otherwise healthy 7-year-old boy is brought to the emergency department because of a 1-day history of involuntary muscle contractions and pain in his back and neck. Two weeks ago, he fell while playing in the sandbox and scraped both his knees. He has not received any vaccinations since birth. His temperature is 38.5°C (101.3°F). He is diaphoretic. Examination shows inability to open his mouth beyond 1 cm. There is hyperextension of the lumbar spine and resistance to neck flexion. Administration of which of the following would most likely have prevented this patient's current condition? a)Capsular polysaccharides b)Chemically-inactivated virus c)Denaturated bacterial product d)Human immunoglobulin against a viral protein e)Viable but weakened microorganism

(Generalized muscle spasms (trismus, mild opisthotonus, nuchal rigidity) and symptoms of autonomic hyperactivity (fever, sweating) following a cutaneous injury in an unvaccinated child suggest tetanus, which is diagnosed clinically) - Denaturated bacterial product: Denatured C. tetani toxoid is used as a vaccination against tetanus. The still intact receptor binding site of the toxoid induces antibody production and memory cell formation (active immunity). Postexposure prophylaxis should include vaccination with DTaP and human tetanus immunoglobulin (TIG), as well as antibiotics and intensive care treatment.

A 55-year-old man comes to the physician because of a 2-month history of headaches, facial numbness, recurrent epistaxis, and a 5-kg (11-lb) weight loss. He recently immigrated from Hong Kong. Examination shows right-sidedcervical lymphadenopathy. Endoscopy shows an exophytic nasopharyngeal mass. Histologic examination of a biopsy specimen of the mass shows sheets of undifferentiated cells with nuclear pleomorphism and abundant mitotic figures. The patient most likely acquired the causal pathogen of his nasopharyngeal mass via which of the following routes of transmission? a)Sexual contact b)Tick bite c)Transfer of saliva d)Fecal-oral e)Blood transfusion f)mother to baby

(Headaches, facial numbness (due to cranial nerve involvement), recurrent epistaxis, weight loss, and lymphadenopathy are suggestive of malignancy. The biopsy of this patient's mass shows undifferentiated nasopharyngeal carcinoma, which is endemicin Southern China. Epstein-Barr virus (EBV) is the primary causative agent in the pathogenesis of this condition) - Transfer of saliva: EBV is mainly transmitted through saliva and respiratory secretions. In addition to nasopharyngeal carcinoma, EBV is associated with the development of infectious mononucleosis ("kissing disease"), oral hairy leukoplakia, primary CNS lymphoma, Hodgkin lymphoma, and Burkitt lymphoma. Most individuals have a latent EBV infection, but malignant complications typically occur only after reactivation in immunocompromised individuals (e.g., patients with HIV).

A 28-year-old woman, gravida 1, para 0, at 32 weeks' gestation comes to the physician for a prenatal visit. She has had no prenatal care. She emigrated from China 5 years ago and cannot recall all of her vaccinations. She appears well. Physical examination shows a uterus consistent in size with a 32-week gestation. Serum studies show: anti hep b surface antibody + anti hep A IgG + Which of the following is the most appropriate initial action by the physician? a)Plan normal vaginal delivery at term b)Determine hepatitis B e antigen serum level c)Plan to give hepatitis B immunoglobulin to newborn

(In this patient, hepatitis viral load would be undetectable.) - normal vaginal delivery: This patient can give a vaginal birth to her baby. Her serum studies are consistent with successful vaccination against hepatitis B, as indicated by her positive anti-hepatitis B surface antibodies, negative hepatitis B core antigen, and negative hepatitis B surface antigen results. Her positive anti-hepatitis A IgG antibody and negative anti-hepatitis A IgM antibody results indicate either HAV vaccination or past infection. Even if the mother had active hepatitis A or B, spontaneous vaginal delivery, rather than C-section, would be recommended.

A 75-year-old woman is brought to the emergency department by her daughter because of shortness of breath and a productive cough with blood-tinged sputum for the past 24 hours. Five days ago, she developed muscle aches, headache, fever, and clear rhinorrhea. These symptoms lasted 3 days. She lives in a house with her daughter. Her temperature is 39.3°C (102.8°F), pulse is 118/min, respirations are 22/min, and blood pressure is 100/60 mm Hg. She appears lethargic. Physical examination shows scattered crackles and rhonchi throughout both lung fields. An x-ray of the chest shows bilateral lobar opacities and several small, thin-walled cystic spaces with air-fluid levelswithin the pulmonary parenchyma. Which of the following is the most likely causal pathogen? a)Mycobacterium tuberculosis b)Pseudomonas aeruginosa c)Staphylococcus aureus d)Legionella pneumoniae e)Respiratory syncytial virus f)Klebsiella pneumoniae g)Streptococcus agalactiae

(High fever, shortness of breath, and productive cough with blood-tinged sputum following a likely episode of influenza (muscle aches, headache, fever, and clear rhinorrhea) are suggestive of necrotizing bronchopneumonia. Thin-walled cysts within the pulmonary parenchyma are consistent with pneumatoceles.) - Staf Aureus: Secondary bacterial pneumonia is a common complication of influenza especially in the elderly, immunocompromised patients, and infants. Patients with secondary bacterial pneumonia classically develop fever and severe respiratory symptoms after initial improvement of influenza symptoms. The most common infective organisms include Streptococcus pneumonia, Staphylococcus aureus, and Haemophilus influenzae. While these pathogens can all potentially cause pneumatoceles, S. aureus is most frequently associated with this finding.

A 38-year-old woman comes to the physician because of a 4-day history of swelling and pain in her left knee. She has had similar episodes of swollen joints over the past 3 weeks. Two months ago, she had a rash on her upper backthat subsided after a few days. She lives in Pennsylvania and works as a forest ranger. Her temperature is 37.8°C (100°F). Physical examination shows a tender and warm left knee. Arthrocentesis of the knee joint yields cloudy fluid with a leukocyte count of 65,000/mm3 and 80% neutrophils. A Gram stain of synovial fluid does not show any organisms. Which of the following is the most likely cause of this patient's condition? a)Production of autoantibodies against Fc portion of IgG b)Infection with spiral-shaped bacteria c)Infection with round bacteria in pairs of two d)Postinfectious activation of innate lymphoid cells of the gut

(If this patient does not receive treatment, she could develop (bilateral) facial nerve palsy and atrioventricular block.) - spiral shaped bacteria: Borrelia burgdorferi is a tick-borne spiral bacteria that causes Lyme disease, which typically begins with a localized rash (i.e., erythema chronicum migrans) and ultimately can cause systemic infection involving the joints, skin, and central nervous system. The incidence of lyme disease is highest in the Northeast and upper Midwest of the US, and outdoor workers (e.g., rangers, landscapers, farmers) are at an increased risk of infection. The nonspecific arthrocentesis findings, i.e., negative Gram stain and leukocytosis with increased neutrophils are consistent with Lyme arthritis. Two-step serological testing is indicated to confirm exposure: The screening test ELISA should be followed by a Western blot test to detect IgG and IgM antibodies against Borrelia. However, positive results only demonstrate exposure to Borrelia, not necessarily current infection. PCR (arthrocentesis) is best for confirming suspected Lyme arthritis in patients who are IgG immunoblot-positive.

A 2-month-old boy is brought to the pediatrician for a routine check-up. His mother says he is feeding well and has no concerns. He is at the 85th percentile for height and 82nd percentile for weight. Immunizations are up-to-date. Results of serum hepatitis B surface IgG antibody testing are positive. Which of the following best explains this patient's hepatitis B virus status? a)Window period b)Chronic infection c)Spontaneous recovery d)Vaccination reaction e)Passive immunity

(IgG is the most common type of antibody and can indicate prior infection or immunization.) - passive immunity: This infant has positive hepatitis B surface IgG antibodies (anti-HBs) acquired via passive immunity from the mother. Since the mother produced anti-HBs, she must have a history of HBV infection or vaccination.

A 23-year-old man comes to the physician because of a 1-week history of muscle ache, fatigue, and fever that occurs every 2 days. He recently returned from a trip to Myanmar. A peripheral blood smear shows erythrocytes with brick-red granules. The physician recommends a combination of two antimicrobial drugs after confirming normal glucose-6-phosphate dehydrogenase activity. Which of the following is the most appropriate rationale for dual therapy? a)Prevention of infection relapse b)Prevention of drug resistance c)Decrease in enzymatic drug deactivation d)Therapy against polymicrobial infections

(In a patient who recently returned from a trip to a malaria-endemic region, the presence of tertian fever and Schuffner granulesin erythrocytes is suggestive of P. vivax or P. ovale, both of which are treated with a combination of chloroquine and primaquine.) - prevention of relapse: Both P. vivax and P. ovale can produce hypnozoites that remain dormant in the liver despite chloroquinetreatment, which only clears infection from the blood. Primaquine is administered to eradicate dormant hypnozoites. G6PD testing is necessary before primaquine or chloroquine therapy because these drugs can trigger a hemolytic crisis in patients with G6PD deficiency.

A 45-year-old man with HIV comes to the physician because of multiple lesions on his chest and lower extremities. The lesions have progressively increased in size and are not painful or pruritic. Current medications include abacavir, dolutegravir, and lamivudine. A photograph of the lesions is shown. His CD4+ T-lymphocyte count is 450/mm3. A skin biopsy shows multiple spindle-shaped cells and lymphocytic infiltrate. Which of the following is the most appropriate pharmacotherapy? a)Alpha-interferon b)Ganciclovir c)Amphotericin B d)doxycycline

(In an HIV-positive patient, multiple painless violaceous skin lesions that progress in size point toward Kaposi sarcoma. Spindle cells and lymphocytic infiltrate on skin biopsy confirm the diagnosis.) - Alpha-interferon (IFN-α) is a biologic modifier that is effective in the treatment of Kaposi sarcoma. This type of sarcoma can be caused by HHV-8 infection and is an AIDS-defining illness. How IFN-α fights cancer cells is not fully understood; upregulation of the immune response is thought to play a role. IFN-α is also used in the treatment of hepatitis B and C infections.

A 67-year-old woman comes to the physician because of fever, chills, myalgias, and joint pain 1 month after undergoing aortic prosthetic valve replacement due to high-grade aortic stenosis. She does not drink alcohol or use illicit drugs. Her temperature is 39.3°C (102.8°F). She appears weak and lethargic. Physical examination shows crackles at both lung bases and a grade 2/6, blowing diastolic murmur over the right sternal border. Laboratory studies show leukocytosis and an elevated erythrocyte sedimentation rate. The causal organism is most likely to have which of the following characteristics? a)Alpha hemolytic, optochin-sensitive diplococci b)Novobiocin-sensitive, coagulase-negative cocci c)Catalase-negative cocci that grows in 6.5% saline d)Alpha hemolytic, optochin-resistant cocci

(In combination with a recent history of aortic valve replacement, this patient's symptoms suggest prosthetic valve endocarditis, the most common cause of which would be Staphylococcus epidermis infection.) - S. epidermidis is a novobiocin sensitive, coagulase-negative, catalase-positive, gram-positive coccus with the capability of biofilm formation. As S. epidermidis is part of the normal skin flora, contamination of prosthetic devices (e.g., heart valves and hip implants) may occur during surgery.

A 2-year-old girl who emigrated from Pakistan 2 weeks ago is brought to the emergency department because of lower limb weakness for one-day. One week ago, she had a 3-day episode of flu-like symptoms that resolved without treatment. She has not yet received any routine childhood vaccinations. Deep tendon reflexes are 1+ in the right lower extremity and absent in the left lower extremity. Analysis of cerebrospinal fluid shows a leukocyte count of 38 cells/mm3 (68% lymphocytes), a protein concentration of 49 mg/dL, and a glucose concentration of 60 mg/dL. Which of the following is the most likely diagnosis in this patient? a)Coxsackie meningitis b)Poliomyelitis c)HSV encephalitis d)Botulism e)Guillain-Barre syndrome

(In this unvaccinated migrant child, the most likely cause of acute flaccid paralysis (weakness with decreased deep tendon reflexes) is anterior horn cell destruction.) - Poliomyelitis, which is caused by the poliovirus, manifests as acute lower motor neuron weakness that begins asymmetrically in the lower extremities and ascends over hours to days. Patients can have pro-dromal flu-like symptoms before the onset of weakness. CSF analysis in poliomyelitis is typically consistent with viral infection (lymphocytic pleocytosis, normal or slightly increased protein levels, and normal glucose levels), as seen here. Diaphragmatic involvement ultimately results in respiratory failure and death.

A 7-year-old boy is brought to the physician by his father because of a 1-day history of a pruritic rash on his trunk and face. Five days ago, he developed low-grade fever, nausea, and diarrhea. Physical examination shows a lace-likeerythematous rash on the trunk and face with circumoral pallor. The agent most likely causing symptoms in this patient has selective tropism for which of the following cells? a)Epithelial cells b)B lymphocytes c)T lymphocytes d)Erythroid progenitor cells

(Infection with this pathogen during pregnancy can lead to hydrops fetalis.) - erythroid progenitor cells: A 7-year-old patient with a pruritic, lace-like erythematous rash on his trunk and face that developed 5 days after a low-grade fever, nausea, and diarrhea most likely has a parvovirus B19 infection. Parvovirus B19 has selective tropism for erythroid progenitor cells. Infection of these cells can result in temporary suspension of erythropoiesis, which leads to transient aplastic crisis.

A 75-year-old woman is brought to the emergency department by her son because of a 2-day history of fever, headache, malaise, and dry cough. The patient lives with her son and his family and her son reports that other members of the family have had similar symptoms during the past week but that he has not had any symptoms. The result of a polymerase chain reaction test confirms that the woman is infected with a virus that has a helical capsidand a segmented genome. Which of the following is the most likely explanation for the son being free of symptoms? a)Vaccination with polyribosylribitol phosphate b)Serum antibodies against hemagglutinin c)Downregulation of ICAM-1 expression d)High concentration of circulating T cells

(Influenza virus is a segmented ssRNA virus with a helical capsid. Children and elderly individuals are at greatest risk for developing severe disease.) - Serum antibodies against hemagglutinin are responsible for immunological protection against influenza virus infections. In the absence of host antibodies against hemagglutinin, influenza virions use hemagglutinin to bind to sialic acid residues on the surface of respiratory epithelial cells and gain entry into the cell. Once inside, the virus can replicate and eventually produce the clinical aspects of the flu. However, in cases of previous exposure to or vaccination against a given strain of influenza, host antibodies against hemagglutinin prevent the virus from entering host cells. The son, therefore, is most likely to have avoided infection as a result of having these neutralizing antibodies.

A 13-year-old girl is brought to the physician because of worsening fever, headache, photophobia, and nausea for 2 days. One week ago, she returned from summer camp. She has received all age-appropriate immunizations. Her temperature is 39.1°C (102.3°F). She is oriented to person, place, and time. Physical examination shows a maculopapular rash. There is rigidity of the neck; forced flexion of the neck results in involuntary flexion of the knees and hips. Cerebrospinal fluid studies show: opening pressure 120 protein 47 clear fluid glucose 68 lymphocytes 85% neutrophils 15% abc 280 Which of the following is the most likely causal organism? a)Mumps virus b)Echovirus c)Listeria monocytogenes d)Streptococcus pneumoniae

(Involuntary flexion of the knees and hips in response to forced flexion of the neck is referred to as a positive Brudzinski sign. Together with fever, headaches, photophobia, and neck stiffness, this suggests acute meningitis. CSF findings of normal opening pressure, clear appearance, slightly elevated protein, normal glucose, and lymphocytic pleocytosis are consistent with aseptic meningitis.) - Echovirus and other enteroviruses are typically responsible for summer outbreaks of aseptic meningitis, especially in children and teenagers. Enteroviruses are the most common cause of aseptic meningitisoverall. Aseptic meningitis commonly manifests with a nonspecific rash, fever, headache, nuchal rigidity, and a positive Brudzinski sign.

A 36-year-old man comes to the physician because of a 2-day history of malaise and a painful, pruritic rash on his lower back and thighs. His temperature is 37.8°C (100°F). Physical examination shows the findings in the photograph. Skin scrapings from the thigh grow neutral colonies on MacConkey agar. The colony-producingbacteria are oxidase-positive. Which of the following is the greatest risk factor for the patient's condition? a)Swimming in pool b)Cat bite c)Unprotected sexual intercourse d)Rose pruning

(MacConkey agar only allows growth of gram-negative organisms. While lactose-fermenting bacteria produce bright pink colonies on this agar, lactose nonfermenters grow in neutral colors. This patient's papulopustular skin eruptions are most likely caused by Pseudomonas aeruginosa, a gram-negative, lactose non-fermenting, oxidase-positive bacillus.) - Swimming in pool: This patient's clinical symptoms (mild fever, pruritic rash, and skin eruptions) are classical findings of hot tub folliculitis, an infection caused by Pseudomonas aeruginosa. While typically dependent on direct inoculation (such as nail puncture wounds) or an immunocompromised host to cause an infection, the most common route of pseudomonal infection in immunocompetent hosts is water (hot tub, lake, pool). Pseudomonal folliculitis is a self-limiting condition, but antibiotic treatment may be warranted in severe cases.

A 21-year-old woman comes to the physician because of a 5-day history of pain with urination and vaginal itching. She is sexually active with multiple partners and uses condoms inconsistently. Pelvic examination shows erythema of the vulva and vaginal mucosa, punctate hemorrhages on the cervix, and green-yellow, malodorous discharge. A photomicrograph of the discharge is shown. Which of the following is the most likely causal organism? a) Neisseria gonorrhoeae b)Treponema pallidum c)Chlamydia trachomatis d)Trichomonas vaginalis

(Microscopic examination of a wet mount preparation of this patient's vaginal discharge shows extracellular, motile, flagellated protozoa) - trichomonad vaginalis: Trichomonas vaginalis infection causes trichomoniasis, which manifests with vaginal pruritus, dysuria, strawberry cervix (punctate hemorrhages on the cervix), and green-yellow, malodorous vaginal discharge as seen in this patient. The presence of motile trophozoites on wet mount preparationconfirms the diagnosis. Vaginal discharge typically has a pH > 4.5. Risk factors for infection include multiple sexual partners and inconsistent condom use.

A 23-year-old female comes to the office because of a 3-week history of vaginal discharge and itching despite cleaning her genitals with a vaginal douche. Her last menstrual period was one week ago. She is sexually active with her new boyfriend. She has an intrauterine device and does not use barrier protection. She was treated for a sore throat infection one month ago. Speculum examination shows erythema around the vaginal introitus and copious white discharge. Vaginal pH is 4.3 and a KOH test shows multiple pseudohyphae on microscopy. Which of the following is the strongest predisposing factor for this patient's condition? a)Intrauterine pregnancy b)Suppression of vaginal bacterial flora c)Increased vaginal pH d)Intrauterine device e)

(Multiple pseudohyphae on wet mount preparation are characteristic of a Candida infection.) - Systemic antibiotic treatment (e.g., oral, intravenous) causes suppression of normal vaginal bacterial flora, which promotes C. albicans overgrowth. Antibiotics increase the incidence of symptomatic vulvovaginitis and vaginal colonization with C. albicans in asymptomatic patients. A normal vaginal pH (< 4.5) and pseudohyphae on microscopic examination of a KOH test confirms the diagnosis. This patient was likely treated with antibiotics for bacterial pharyngitis.

A 47-year-old woman comes to the physician because of a 6-week history of fatigue and low-grade fever. She has no history of serious illness except for a bicuspid aortic valve, diagnosed 10 years ago. She does not use illicit drugs. Her temperature is 37.7°C (99.9°F). Physical examination shows petechiae under the fingernails and multiple tender, red nodules on the fingers. A new grade 2/6 diastolic murmur is heard at the right second intercostal space. Which of the following is the most likely causal organism? a)enterococcus b)staph epidermidis c)strep pyogenes d)strep sanguinis e)staph aureus

(Osler nodes, subungual hemorrhage, and a new murmur in a patient with a 6-week history of fatigue and low-grade feversuggest subacute infective endocarditis. In patients with an underlying valve disease, infection is typically caused by a gram-positive, catalase-negative, coagulase-negative, α-hemolytic, optochin-resistant coccus) - Streptococcus sanguinis: S. sanguinis is a gram-positive member of the viridans streptococci group, which are oral commensalbacteria. Further viridans streptococci include S. mutans and S. mitis. Viridans streptococci are the most common cause of subacute endocarditis in predamaged native valves (such as this patient's bicuspid aortic valve) and endocarditis in prosthetic valves occurring > 60 days post-operation. Viridans streptococci are also a common cause of endocarditis after oral procedures, which frequently lead to transient bacteremia. In patients with predamaged native valves, circulating bacteria may subsequently adhere to and infect platelet-fibrin aggregates that form at sites of endocardial injury, leading to vegetation formatio

Three weeks after undergoing transurethral prostate resection for benign prostatic hyperplasia, a 70-year-old man has fever, malaise, and pain in his extremities. Physical examination shows subungal petechiae and tender red papules on his fingers and toes. A new holosystolic murmur is heard on chest auscultation. A photomicrograph of a Gram stain of an isolate from blood culture is shown. The organism does not cause hemolysis on blood agar. Addition of pyrrolidonyl-β-naphthyide gives the bacterial colonies a cherry red color. Which of the following is the most likely causal organism? a)Staphylococcus aureus b)Enterococcus faecalis c)Pseudomonas aeruginosa d)Streptococcus mutans

(Presence of a new holosystolic murmur, Osler nodes, and splinter hemorrhages is suggestive of infective endocarditis. Photomicrograph and blood agar culture reveal gram-positive cocci that are γ-hemolytic and PYR-positive.) - Enterococcus faecalis: Enterococcus faecalis is a gram-positive, PYR positive, catalase-negative coccus that can grow in 6.5% NaCl and produces variable hemolysis (α-hemolytic or γ-hemolytic) on blood agar. It can cause subacute infective endocarditis in patients who undergo gastrointestinal or genitourinary procedures (e.g., transurethral prostate resection). Enterococcus spp. are resistant to drugs typically used to treat infective endocarditis such as penicillin G, 1st to 4th generation cephalosporins, and vancomycin (in the case of VRE). Enterococcus is sensitive to aminopenicillins and 5th generation cephalosporins (e.g., ceftaroline).

An investigator is developing a drug that selectively inhibits the retrograde axonal transport of rabies virus towards the central nervous system. To achieve this effect, this drug must target which of the following? a) Tubulin b)Kinesin c)Dynein d)Nidogen e)Acetylcholine

(Rabies virus travels from the site of inoculation to the central nervous system (CNS) with the aid of motor proteins that facilitate the movement of cargo proteins across axons.) - Dynein is an ATP-dependent motor protein responsible for the retrograde migration and movement of cellular organelles and proteins along microtubules in axons. After rabies exposure, the rabies virus binds to acetylcholine receptors and enters the motor and sensory nerves of the peripheral nervous system, spreading in a retrograde direction to the CNS. To inhibit the migration of the virus and subsequent brain infection, the new drug must therefore target dynein motors.

A 3255-g (7-lb) female newborn is delivered at term. Pregnancy and delivery were uncomplicated. On the day of her birth, she is given a nonconjugated routine childhood vaccine that contains a noninfectious glycoprotein. This vaccine will most likely help prevent infection by which of the following pathogens? a)Haemophilus influenzae type b b)Hepatitis D virus c)poliovirus d)rotavirus

(The antigen in this vaccine protects against two pathogens simultaneously.) - Infection by hepatitis D virus (HDV) can be prevented by hepatitis B vaccination. On its own, HDV is "defective" and unable to cause disease. HDV requires the hepatitis B surface antigen (HBsAg) for replication (viral complementation). HBsAg, a noninfectious glycoprotein, is the antigen used to make the hepatitis B vaccine. Hepatitis B is spread through sexual activity, via needle sharing, or at birth from the mother to child.

A 2-year-old boy is brought to the physician because of fever, productive cough, and shortness of breath. Since birth, he has had multiple respiratory infections requiring treatment with antibiotics. His immunizations are up-to-date. He is in the 10th percentile for height and weight. His temperature is 38°C (100.3°F). Examination detects diffuse bilateral wheezing and cervical lymphadenopathy. Flow cytometric analysis of a serum sample from the patient fails to fluoresce after incubation with dihydrorhodamine. This patient is at greatest risk of infection with which of the following organisms? a)Enterococcus faecium b)Actinomyces israelii c)Serratia marcescens d)Streptococcus pneumoniae e)Streptococcus pyogenes

(Recurrent bacterial infections in a child raise suspicion of a congenital immunodeficiency. The abnormal dihydrorhodamine flow cytometry test indicates that this patient's neutrophils cannot mount a normal respiratory burst by generating superoxideradicals from oxygen, which is diagnostic of chronic granulomatous disease (CGD). Affected patients are particularly susceptible to infection with catalase-positive organisms.) - Serratia marcescens is a catalase-positive, gram-negative bacillus that commonly affects patients who have CGD. In CGD, neutrophils are unable to generate superoxide radicals from oxygen. Consequently, host neutrophils rely on the hydrogen peroxide produced by bacteria as a precursor to bactericidalreactive oxygen species. Catalase-producing organisms such as Serratia marcescens, however, are able to metabolize their own hydrogen peroxide. This deprives CGD host neutrophils of the substrate needed to form reactive oxygen species, predisposing patients to infection with such bacteria.

A 22-year-old man comes to the physician because of abdominal pain, diarrhea, and weight loss that started after a recent backpacking trip in Southeast Asia. He does not smoke or drink alcohol. His leukocyte count is 7,500/mm3(61% segmented neutrophils, 13% eosinophils, and 26% lymphocytes). Stool microscopy shows rhabditiform larvae. This patient is most likely to develop which of the following? a)Muscle tenderness b)Rectal prolapse c)Perianal serpiginous rash d)Hematuria

(Rhabditiform larvae on stool microscopy, eosinophilia, and a recent travel history to Southeast Asia are consistent with Strongyloides stercoralis (threadworm) infection.) - perianal rash: Strongyloides stercoralis is a nematode endemic to Southeast Asia that is transmitted via larval penetration of intact skin (usually when bare feet come into contact with contaminated soil). Larvae travel to the alveoli through the bloodstream, where they ascend the pulmonary system before being coughed up and swallowed. Once in the GI tract, they become adult female worms and reproduce via parthenogenesis, with the eggs hatching as rhabditiform larvae. Rhabditiform larvae then become infective filariform larvae, penetrating intestinal mucosa or the skin of the perianal region, which completes the process of autoinfection. This results in a rapidly migrating (5 cm or more per hour) pink, raised track, referred to as larva currens; this is pathognomonic of Strongyloides infection.

Three days after surgical repair of a distal right radius fracture, a 62-year-old man develops severe wound pain and fever. His temperature is 39.1°C (102.4°F). Physical examination shows erythema and edema of the right arm that extends up to the elbow. Yellow pus drains from the surgical wound. Cultures from the wound fluid grow gram-positive bacteria in grape-like clusters that are resistant to nafcillin. While treating the patient's condition, the hospital also wants to eradicate the causal pathogen in asymptomatic carriers. Which of the following measures would be most effective? a)Alcohol-based oral rinse b)Permethrin hair wash c)Colistin nebulized aerosol d)Vancomycin tablet e)Chlorhexidine hand disinfection f)Mupirocin nasal ointment

(Staphylococcus aureus is a gram-positive bacterium that grows in grape-like clusters and often causes nosocomial wound infections. Resistance to nafcillin indicates the growth of methicillin-resistant S. aureus (MRSA). - Mupirocin ointment applied to the nares is the treatment of choice for MRSA decolonization. The anterior nares are the most common site of MRSA colonization; other sites include the hands, axillae, and perineum. MRSA infections are usually preceded by an asymptomatic carrier state. In addition to intranasal mupirocin therapy, regular washes with chlorhexidine solution are also recommended.

A 23-year-old woman comes to the physician because of vaginal discharge for 4 days. Her last menstrual period was 3 weeks ago. Twelve months ago, she was diagnosed with trichomoniasis, for which she and her partner were treated with a course of an antimicrobial. She is sexually active with one male partner, and they use condoms inconsistently. Her only medication is a combined oral contraceptive that she has been taking for the past 4 years. A Gram stain of her vaginal fluid is shown. Which of the following is the most likely causal organism? a)Neisseria gonorrhoeae b)Gardnerella vaginalis c)Haemophilus ducreyi d)Chlamydia trachomatis

(The Gram stain shows gram-negative intracellular diplococci - the pathogen responsible for a common sexually transmitted infection (STI). - Neisseria gonorrhoeae is responsible for gonorrhea, an STI that presents with purulent, creamy vaginal discharge. This patient's history of inconsistent condom use and previous trichomoniasis indicates an elevated risk of future STIs. The presence of gram-negative intracellular diplococci in the microscopy of this patient's discharge specimen confirms that the infection is caused by N. gonorrhoeae.

A 39-year-old man comes to the emergency department because of a 2-day history of fever, chills, dyspnea, and a non-bloody productive cough. He was diagnosed with HIV infection 4 years ago and has been on highly active antiretroviral therapy since then. His temperature is 38.8°C (101.8°F). Examination shows crackles over the left lower lung base. His CD4+ T-lymphocyte count is 520/mm3. An x-ray of the chest shows an infiltrate in the left lower lobe. Sputum cultures grow colonies with a narrow zone of green hemolysis without clearing on blood agar. The most likely causal pathogen of this patient's condition produces which of the following virulence factors? a)M protein b)Beta-glucan cell wall c)Lipopolysaccharide d)Polysaccharide capsule

(The acute onset of symptoms (fever, chills, dyspnea and productive cough), crackles on auscultation, and findings on x-rayconfirm lobar pneumonia. A normal CD4+ T-lymphocyte count makes an opportunistic infection unlikely. The most common cause of community-acquired pneumonia (CAP) is Streptococcus pneumoniae, an alpha-hemolytic coccus, which is consistent with this patient's sputum cultures.) - The most important virulence factor of Streptococcus pneumoniae is its polysaccharide capsule, which masks subcapsular antigens. The capsule prevents opsonization (inhibits binding of antibodies and complement activity), which protects bacteria from phagocytosis (e.g., by macrophages in the spleen). Conjugate vaccines against S. pneumoniae are directed against capsular polysaccharide and proteins and are particularly important in patients with anatomic or functional asplenia. Once the host's plasma cellsproduce antibodies against the polysaccharide capsule, the infection with S. pneumoniae can be fought more effectively.

A 26-year-old woman comes to the emergency department with fever, abdominal pain, and nausea for the past 7 hours. The pain started in the right lower abdomen but has now progressed to diffuse abdominal pain. Her temperature is 39.5°C (103.1°F). Physical examination shows generalized abdominal tenderness with rebound, guarding, and decreased bowel sounds. She is taken for an emergency exploratory laparoscopy, which shows a perforated appendix with an adjacent abscess and peritoneal inflammation. Cultures from the abscess fluid grow catalase-producing, anaerobic, gram-negative rods that have the ability to grow in bile. Which of the following is the most appropriate pharmacotherapy for this patient? a)Vancomycin and azithromycin b)Gentamicin c)Piperacillin d)Cefazolin and doxycycline e)Ampicillin and sulbactam

(The bacterial cultures suggest Bacteroides fragilis infection. Bacteroides fragilis is a normal part of the intestinal flora but can cause peritonitis and abscess formation.) - Ampicillin and sulbactam: A patient with perforated appendicitis and abscess formation requires antibiotics with anaerobic and gram-negative coverage. Because more than 90% of B. fragilis strains produce beta-lactamase, ampicillinshould be combined with the beta-lactamase inhibitor sulbactam. Other indications for ampicillin/sulbactam therapy include skin/soft tissue infections caused by gram-positive organisms, including Staphylococcus aureus (but not MRSA) and complicated pelvic/gynecological infections.

A 24-year-old female comes to the physician because of flu-like symptoms and a new rash for 2 days. She denies contacts with sick individuals or recent travel abroad, but recently went camping in Vermont. Vital signs are within normal limits. Examination of the lateral right thigh shows a circular red ring with central clearing. Which of the following is the natural reservoir of the pathogen responsible for this patient's symptoms? a)rat b)mouse c)tick d)flea

(The circular bullseye rash (erythema migrans) and flu-like symptoms seen in this patient after a recent camping trip raise concern for Lyme disease.) - mouse: In the US, the primary reservoir host for Borrelia burgdorferi, the pathogen responsible for Lyme disease, is the white-footed mouse (Peromyscus leucopus). Another host is the white-tailed deer. The Ixodes tickis the main vector responsible for transmission to humans.

A 51-year-old man with alcohol use disorder comes to the physician because of a fever and productive cough. An x-ray of the chest shows a right lower lobe consolidation and a diagnosis of aspiration pneumonia is made. The physician prescribes a drug that blocks peptide transfer by binding to the 50S ribosomal subunit. Which of the following drugs was most likely prescribed? a) doxycycline b)metronidazole c)levofloxacin d)clindamycin e)ceftriaxone

(The drug in question is a bacteriostatic antibiotic frequently used to cover anaerobic bacteria.) -Clindamycin:Clindamycin is a bacteriostatic antibiotic that inhibits bacterial protein synthesis by preventing peptidetranslocation at the 50S (large) ribosomal subunit. It targets anaerobic organisms and is commonly used for the treatment of aspiration pneumonia and lung abscess, which are typically polymicrobial. Clostridioides difficile colitis is a potential complication of clindamycin.

An investigator is studying the mechanism of HIV infection in cells obtained from a human donor. The effect of a drug that impairs viral fusion and entry is being evaluated. This drug acts on a protein that is cleaved off of a larger glycosylated protein in the endoplasmic reticulum of the host cell. The protein that is affected by the drug is most likely encoded by which of the following genes? a)rev b)gag c)pol d)env e)tat

(The drug that is being evaluated is most likely enfuvirtide, which binds glycoprotein 41 (gp41) to inhibit viral fusion and entry. Gp41, along with gp120, is formed after cleavage of gp160 in the endoplasmic reticulum of an HIV-infected host cell.) - The structural gene env encodes for the proteins gp120 and gp41. These two proteins are required for initial infection and entry into host cells by the HIV virion. The virus attaches to CD4+ receptors on T cells via its gp120 glycoprotein, while fusion and entry are mediated via the gp41 glycoprotein. These vital proteins have become a target for drugs like enfuvirtide, a fusion inhibitor that competitively binds to the viral protein gp41 and thereby prevents fusion of HIV with its target cell.

A 21-year-old man comes to the physician because of painful, firm, dark bumps on his neck and jawline. He has no history of serious illness and takes no medications. His brother had a similar rash that improved with topical erythromycin therapy. A photograph of the rash is shown. Which of the following is the most likely underlying mechanism of this patient's condition? a)Trichophyton infection of the superficial hair follicle b) Cutibacterium acnes colonization of the pilosebaceous unit c)Follicular obstruction with subsequent duct rupture d) Interfollicular penetration of the skin by distal end of hair

(The image shows multiple small, dark, firm papules and some pustules on a man's shaved neck, which are manifestations of pseudofolliculitis barbae) - Interfollicular penetration of the skin by distal end of hair: Pseudofolliculitis barbae primarily affects African-American men and is characterized by an inflammatory reaction to ingrown hairs (e.g., after shaving). The hair either enters the interfollicular epidermis after exiting the follicular orifice (extrafollicular penetration) or penetrates the dermis before exiting the follicular orifice (transfollicular penetration). The resulting inflammatory reaction against the hair leads to the characteristic finding of painful papules distributed along the areas of facial hairgrowth. Management of pseudofolliculitis barbae includes cessation of shaving and adjunctive topical therapies (e.g., topical corticosteroids to reduce inflammation and antimicrobial therapy to treat secondary infection).

An investigator studying influenza virus variability isolates several distinct influenza virus strains from the respiratory secretions of a study subject. Mass spectrometry analysis of one strain shows that it expresses neuraminidase on its surface. Subsequent sequencing of this strain shows that its genome lacks the neuraminidase gene. Which of the following is the most likely explanation for this finding? a)Transduction b)Reassortment c)Recombination d)Phenotypic mixing

(The influenza virus strain under investigation has surface neuraminidase without a genome encoding for the protein. It must have therefore acquired the surface protein from one of the other coinfecting influenza viruses.) - Phenotypic mixing is the process by which a virus retains its own genetic material but becomes coated with surface proteins of a different coinfecting virus. This is observed in the influenza virus studied here, which expresses neuraminidase without having the actual genetic code. In some cases, phenotypicmixing can change a noninfectious virus into an infectious strain, as surface proteins determine host tropism. However, the surface protein will not be expressed in the subsequent progeny because it is not encoded in the viral genome.

An investigator is studying growth patterns of various fungal pathogens. Incubation of an isolated fungus at 25°C shows branching hyphae with rosettes of conidia under light microscopy. After incubation at 37°C, microscopic examination of the same organism instead shows smooth, white colonies with rounded, elongated cells. Infection with the investigated pathogen is most likely to cause which of the following conditions? a)Cryptococcosis b)Pityriasis versicolor c)Mucormycosis d)Aspergillosis e)Candidiasis f)Coccidioidomycosis g)Sporotrichosis

(The isolated organism exhibits fungal dimorphism. One of the characteristic clinical features in infected patients is ascending lymphangitis) - Sporotrichosis: Sporothrix schenckii, which causes sporotrichosis, is most likely the fungus in question. As a dimorphic fungus, Sporothrix exists in yeast or hyphal forms depending on the surrounding temperature. At 25°C, it takes on its hyphal form, with septate branching hyphae and oval conidia. Clusters of these conidiamay appear as rosettes. However, when grown at 37°C, microscopic evaluation reveals a cigar-shaped budding yeast. Infection occurs after a penetrating skin injury (e.g., from a thorn prick, explaining its prevalence in gardeners and similar workers) and initially manifests as a pustule or ulcer that progresses to ascending lymphangitis.

An investigator is studying the growth of an organism in different media. The organism is inoculated on a petri dish that contains heated sheep blood, vancomycin, nystatin, trimethoprim, and colistin. The resulting growth medium is incubated at 37°C. Numerous small, white colonies are seen after incubation for 48 hours. This organism is most likely to cause which of the following conditions? a)Diphtheria b)Hemolytic uremic syndrome c)Oral thrush d)Gonorrhea

(The medium described is Thayer-Martin agar, which contains antimicrobial drugs that inhibit the growth of gram-positivebacteria (vancomycin), many gram-negative bacteria (colistin, trimethoprim), and fungi (nystatin), which favors the selective growth of one particular bacterial genus.) - Gonorrhea is caused by Neisseria gonorrhoeae. Unlike other gram-negative bacteria, Neisseria spp. are resistant to trimethoprim and colistin and thus selectively grow on Thayer-Martin agar.

A 3-month old male infant with HIV infection is brought to the physician for evaluation. The physician recommends monthly intramuscular injections of a monoclonal antibody to protect against a particular infection. The causal pathogen for this infection is most likely transmitted by which of the following routes? a)Aerosol inhalation b)Breast feeding c)Blood transfusion d)Fecal-oral route

(The only FDA-approved monoclonal antibody that is currently available for infection prophylaxis is palivizumab. It prevents infection by an enveloped virus with a helical capsid and single-stranded, negative-sense, non-segmented RNA.) - Regular administration of monoclonal antibody palivizumab is recommended for patients in immunocompromised states (e.g., HIV infection). It prevents infection with respiratory syncytial virus(RSV), which may cause bronchiolitis in infants. RSV is usually transmitted via aerosol inhalation.

A 64-year-old female with type 2 diabetes mellitus comes to the physician because of a 1-week history of painful red swelling on her left thigh. Examination shows a 3- x 4-cm, tender, fluctuant mass. Incision and drainage of the abscess are performed. Culture of the abscess fluid grows gram-positive, coagulase-positive cocci that are resistant to oxacillin. Which of the following best describes the mechanism of resistance of the causal organism to oxacillin? a)Degradation of the antibiotic b)Decreased uptake of the antibiotic c)Acetylation of the antibiotic d)Altered target of the antibiotic

(The organism that most likely caused the abscess in this patient is Staphylococcus aureus.) - altered target of the ab: The mechanism of resistance to penicillins and cephalosporins is usually attained by degradation of the antibiotic by a beta-lactamase and/or alteration of the antibiotic binding site. Oxacillin is a penicillinase-resistant penicillin (i.e., it is not susceptible to beta-lactamases); the development of resistance to this antibiotic is therefore due to alteration of the penicillin-binding protein binding site, leading to a reduced affinity and reduced inactivation of the pathogen. This is one of the main virulence factors in MRSA, which has allowed this microorganism to become resistant to more classes of antibiotics, including penicillinase-resistant penicillins (e.g., oxacillin, dicloxacillin, and nafcillin).

An investigator is studying the genetic profile of an isolated pathogen that proliferates within macrophages. The pathogen contains sulfatide on the surface of its cell wall to prevent fusion of the phagosome and lysosome. She finds that some of the organisms under investigation have mutations in a gene that encodes the enzyme required for synthesis of RNA from a DNA template. The mutations are most likely to reduce the therapeutic effect of which of the following drugs? a)Streptomycin b)Rifampin c)Levofloxacin d)Ethambutol e)Amikacin

(The pathogens in question are likely mycobacteria. An antimycobacterial drug that targets nucleic acid polymerization would not be very effective against these organisms.) - Rifampin is a noncompetitive inhibitor of DNA-dependent RNA-polymerase. Mutations in the polymerase gene are a major mechanism of acquired drug resistance and most commonly occur when rifampin is used as monotherapy. For this reason, rifampin is most often prescribed as part of a multidrug regimen (along with isoniazid, pyrazinamide, and ethambutol) to treat active infection with Mycobacterium tuberculosis. It is also used as prophylaxis for individuals who have been exposed to meningococcus or H. influenzae type B.

A 4-year-old girl is brought to the physician because of diarrhea and vomiting for 5 days. Vaccinations are up-to-date. She appears pale and irritable. Her vital signs are within normal limits. Examination shows petechiae on her trunk and extremities. Abdominal examination shows diffuse abdominal tenderness with hyperactive bowel sounds. The remainder of the exam shows no abnormalities. Laboratory studies show: platelets 49,000 LDH 300 creatinine 1.6 Coagulation studies are normal. A peripheral blood smear is shown. Which of the following is the most likely underlying cause of these findings? a)Acute lymphocytic leukemia b)Immune thrombocytopenic purpura c)Disseminated intravascular coagulation d)Escherichia coli infection e)Parvovirus B19 infection

(The schistocytes seen on this patient's blood smear, as well as her recent diarrheal illness, suggest a certain diagnosis) - Escherichia coli infection: This child presents with the triad of thrombocytopenia, signs of hemolysis (anemia, ↑ LDH, schistocytes), and renal dysfunction, which is indicative of hemolytic uremic syndrome (HUS). The Shiga-toxin-producing E. coli (STEC) is the most common cause of pediatric HUS. The illness is often preceded by a prodrome of vomiting and diarrhea, decreased urine output, and lethargy.

A 54-year-old man comes to the physician because of persistent right knee pain and swelling for 2 weeks. Six months ago, he had a total knee replacement because of osteoarthritis. His temperature is 38.5°C (101.3°F), pulse is 100/min, and blood pressure is 139/84 mm Hg. Examination shows warmth and erythema of the right knee; range of motion is limited by pain. His leukocyte count is 14,500/mm3, and erythrocyte sedimentation rate is 50 mm/hr. Blood cultures grow gram-positive, catalase-positive cocci. These bacteria grow on mannitol salt agar without color change. Production of which of the following is most important for the organism's virulence? a)Protein A b)Vi capsule c)Exotoxin A d)M protein e)Exopolysaccharides

(The patient's blood culture shows gram-positive, catalase-positive cocci that do not ferment mannitol (i.e., do not produce a color change on mannitol salt agar), which is characteristic of Staphylococcus epidermidis.) - Exopolysaccharides are a primary component of biofilm, which is the major virulence factor of Staphylococcus epidermidis, a part of the natural skin flora and one of the leading causative pathogens of foreign body-related infections. Biofilm consists of an extracellular matrix of polysaccharides and protein, which both adheres S. epidermidis to the foreign material and protects the bacteria from host defenses and antimicrobial agents. The typical presentation of infection with S. epidermidis is a delayed-onset, highly resistant infection in patients with indwelling medical devices (e.g., prosthetic orthopedic devices, intravascular catheters, and cerebral shunts), as seen here. Other bacteria that use biofilmsinclude Pseudomonas aeruginosa and viridans streptococci.

A 59-year-old man is brought to the emergency department by his wife because of fever, chills, night sweats, and generalized fatigue for 2 weeks. His temperature is 39.1°C (102.4°F). He appears ill. Physical examination shows a grade 3/6 mid-diastolic murmur at the left sternal border, and crackles at both lung bases. A transesophageal echocardiography shows a 12 mm vegetation on the aortic valve. Blood cultures show gram-positive, catalase-negative, gamma-hemolytic cocci in chains that are unable to grow in a 6.5% NaCl medium. Which of the following is the most likely predisposing factor for this patient's current condition? a)Periodontal disease b)Colon cancer c)Urinary tract infection d)Congenital heart defect

(The patient's blood cultures and echocardiography findings are consistent with aortic regurgitation (diastolic murmur at the left sternal border) due to Streptococcus gallolyticus endocarditis.) - colon cancer: S. gallolyticus can be found in the normal bowel flora of healthy humans. There is a well-established link between S. gallolyticus bacteremia (particularly in endocarditis) and colorectal cancer. However, it remains unclear whether S. gallolyticus is part of the etiology of colorectal malignancies or a consequence of the disease. Animal studies suggest that this pathogen may play a role in the pathogenesis of colorectal malignancies. A colonoscopy is indicated in all patients with S. gallolyticus on blood culture.

A 16-year-old girl is brought to the physician because of a 1-month history of fever, headaches, and profound fatigue. Her temperature is 38.2°C (100.8°F). Examination shows splenomegaly. Laboratory studies show: heterophile antibody - ebv antibodies - In an immunocompromised host, the causal organism of this patient's symptoms would most likely cause which of the following conditions? a)Multiple cerebral abscesses with surrounding edema b)Non-scrapable white patches on the lateral tongue c)Linear ulcers near the lower esophageal sphincter d)Diffuse pulmonary infiltrates with pneumatoceles

(The patient's constitutional symptoms, fever, splenomegaly, and the finding of atypical lymphocytosis with ≥ 10% lymphocytesand mild elevation of transaminases are characteristic of mononucleosis. The absence of exudative pharyngitis and lymphadenopathy, in combination with the negative monospot test (heterophile antibody assay) and negative EBV VCA antibodies, make cytomegalovirus (CMV) the most likely causal pathogen.) - linear ulcers in esophagus: Although CMV is asymptomatic in the majority of cases involving immunocompetent individuals, it can cause CMV mononucleosis in up to 10% of the population. In contrast, CMV infection in immunocompromised individuals can cause CMV esophagitis, manifesting as linear ulcers near the lower esophageal sphincter and many other complications, including CMV pneumonia, CMV retinitis, and CMV encephalitis (all of which are AIDS-defining conditions).

A 5-year-old boy is brought to the physician because of intermittent abdominal cramps and recurrent episodes of foul-smelling greasy diarrhea for 3 months. He has a history of recurrent upper respiratory infection. The abdomen is diffusely tender to palpation and resonant to percussion. A photomicrograph of a stool sample is shown. This patient is at increased risk for which of the following? a)Hypersensitivity reaction to transfusion b)Disseminated tuberculosis c)Gastric adenocarcinoma d)Progressive peripheral neuropathy

(The photomicrograph shows an egg-shaped cell with a two-layered cell wall, multiple nuclei, and longitudinal threadlike-structures called median bodies. These are the typical characteristics of Giardia lamblia cysts. Recurrent sinopulmonary (mostly caused by encapsulated bacteria) and GI infections (e.g., giardiasis) are typical in IgA deficiency, as secretory IgA antibodies are a crucial part of mucosal defense.) - hypersensitivity reaction to transfusion: Patients with IgA deficiency may develop anti-IgA antibodies. Any transfusion that contains even trace amounts of IgA (e.g., RBC transfusions) can cause an anaphylactic transfusion reaction in such patients. If transfusions are necessary, the patient should first be screened for anti-IgA antibodies and transfusionshould be prepared from another IgA-deficient individual. If this is not possible and the patient does have documented anti-IgA antibodies, saline-washed RBC transfusions can be used.

A 49-year-old man comes to the physician for evaluation of several painless, pruritic lesions on his left forearm that he first noticed 4 days ago. They were initially pink marks that progressed into blisters before ulcerating. He has also had a headache for 1 week. His temperature is 38.1°C (100.6°F). A photograph of one of the lesions is shown. There is pronounced edema of the surrounding skin and painless swelling of the left axillary lymph nodes. Which of the following is the greatest risk factor for this patient's condition? a)Pool swimming b)Drug exposure c)Cat scratch d)Wool handling e)Sexual contact

(The picture shows a black eschar characteristic of cutaneous anthrax.) - Wool handling is the greatest risk factor for cutaneous anthrax, which was traditionally known as woolsorter's disease. Human infection with Bacillus anthracis usually occurs as a result of contact with infected animals or infected animal products. The lesions typically begin as papules that expand to become eschars surrounded by extensive edema and lymphadenopathy. The disease can be accompanied by systemic symptoms such as fever, malaise, and headache and rarely progresses to bacteremia or death.

A 16-year-old girl is brought to the emergency department by her parents because of fever, vomiting, rash, and worsening confusion since this morning. On questioning, her mother reports that her last menstrual period was 1 week ago and that she recently started using tampons. She appears lethargic and is only oriented to person. Her temperature is 40.4°C (104.7°F), pulse 174/minute, and blood pressure is 62/44 mm Hg. Examination shows oropharyngeal hyperemia and diffuse macular erythroderma. Which of the following is the most likely cause of this patient's condition? a)Erythrogenic toxin production b)Lipooligosaccharide expression c)Unregulated B cell proliferation d)Generalized mast cell degranulation e)Polyclonal T cell activation

(The sudden onset of high fever, diffuse macular erythroderma, confusion, and shock (hypotension and tachycardia) in a young girl who recently had her menstrual period raises concern for toxic shock syndrome (TSS) as a result of prolonged tampon use.) - Polyclonal T cell activation: Toxic shock syndrome (TSS) is a life-threatening systemic inflammatory reaction caused by bacterial exotoxins, e.g., toxic shock syndrome toxin-1 (TSST-1) produced by Staphylococcus aureus. TSST-1 is a superantigen that triggers polyclonal T cell activation by binding to MHC II and T-cell receptors, resulting in the massive release of IL-2, IFN-γ, and TNF-α. This leads to the typical symptoms of high fever, rash, altered mental status, and shock. About half of TSS cases are caused by prolonged use of tampons over several days.

An investigator studying mechanisms of acquired antibiotic resistance in bacteria conducts a study using isolated strains of Escherichia coli and Staphylococcus aureus. The E. coli strain harbors plasmid pRK212.1, which conveys resistance to kanamycin. The S. aureus strain is susceptible to kanamycin. Both bacterial strains are mixed in a liquid growth medium containing deoxyribonuclease. After incubation for 2 days and subsequent transfer to a solid medium, the S. aureus colonies show no lysis in response to the application of kanamycin. Analysis of chromosomal DNA from the kanamycin-resistant S. aureus strain does not reveal the kanamycin-resistance gene. Which of the following mechanisms is most likely responsible for this finding? a)Transformation b)Conjugation c)Transposition d)Secretion e)Transduction

(The transfer of the pRK212.1 kanamycin-resistance plasmid from one bacterium to another (horizontal gene transfer) was most likely achieved via the production of a sex pilus.) - Bacterial conjugation, the transfer of genetic material from one bacterium to another across a sex pilus(mating bridge), is a mechanism for the horizontal transfer of antibiotic resistance genes. The resistance-conferring plasmid (R-plasmid) usually also contains F (fertility) factor genes, which enable the donor bacterium to form a sex pilus. In F+ x F- conjugation, as seen here, a single strand of the R-plasmid is transferred to the recipient, while the complementary strand remains in the donor. Both bacteria then make complimentary copies of the single-stranded R-plasmid, which makes them resistant to the antibiotic as well as capable of producing sex pili and transferring the R-plasmid to other bacteria. In Hfr x F- conjugation, the R-plasmid is first incorporated into a region of the donor bacterium's chromosometo form a high-frequency recombination (Hfr) cell. Subsequently, a part of a single strand of the R-plasmid along with some flanking bacterial chromosomal DNA is transferred to the recipient, which can acquire antibiotic resistance in this manner but is incapable of producing sex pili.

A 48-year-old patient comes to the physician because of a 4-day history fever, headaches, loss of appetite, and myalgia. Two weeks ago, the patient went on a camping trip to Wisconsin. His temperature is 39.5°C (103.1°F). Physical examination shows no rash or joint swelling. Laboratory studies show: leukocyte 3,800 platelet 85,000 AST 68 ALT 72 Treatment with doxycycline is begun and the patient recovers. A paired acute and convalescent indirect fluorescent antibody test confirms that the patient had anaplasmosis. The patient's condition was most likely transmitted by which of the following vectors? a)Louse b)Lone star tick c)Ixodes tick d)Flea

(The vector responsible for the patient's condition is also a vector for Lyme disease and babesiosis.) - Ixodes ticks are vectors for anaplasmosis, Lyme disease, and babesiosis. Ixodes scapularis is found in the northeast and midwest US (e.g., Wisconsin), while Ixodes pacificus is found primarily along the west coast of the US. Anaplasmosis is caused by the bacterium Anaplasma phagocytophilum and typically manifests with flu-like and gastrointestinal symptoms within 1-2 weeks after the bite of an infected tick. Typical laboratory findings include leukopenia, thrombocytopenia, and elevated transaminaselevels, all of which are seen in this patient.

An investigator is studying bacterial toxins in a nonpathogenic bacterial monoculture that has been inoculated with specific bacteriophages. These phages were previously cultured in a toxin-producing bacterial culture. After inoculation, a new toxin is isolated from the culture. Genetic sequencing shows that the bacteria have incorporated viral genetic information, including the gene for this toxin, into their genome. The described process is most likely responsible for acquired pathogenicity in which of the following bacteria? a)Staphylococcus aureus b)Corynebacterium diphtheriae c)Haemophilus influenzae d)Streptococcus pneumoniae

(The virus has infected the previously nonpathogenic bacteria and incorporated its own viral genetic information into the bacterial genome, giving the bacteria a new virulence factor. This process is called specialized transduction.) - Nontoxigenic strains of Corynebacterium diphtheriae can acquire the gene that encodes the diphtheria toxin through specialized transduction. This occurs when a toxigenic C. diphtheriae is infected with a lysogenic bacteriophage and the viral genome is inserted into the bacterial genome. As the phagereplicates and excises its own nucleic acids from the bacterial genome to produce new phages, the flanking bacterial toxin gene is also copied to the new viruses. Any subsequent nontoxigenic bacteria infected with these phages will then acquire a copy of the toxin gene, which has become part of the phage genome. The genes for many other bacterial toxins such as erythrogenic exotoxin A, botulinum toxin, cholera toxin, and Shiga toxin are also transferred by specialized transduction.

Six days after undergoing an elective hip replacement surgery, a 79-year-old man develops dysuria, flank pain, and fever. His temperature is 38.5°C (101.3°F). Examination shows marked tenderness in the right costovertebral area. Treatment with an antibiotic is begun, but his symptoms do not improve. Further evaluation shows that the causal organism produces an enzyme that inactivates the antibiotic via phosphorylation. An agent from which of the following classes of antibiotics was most likely administered a)ahminoglycosides b)tetracyclines c)cephalosporins d) glycopeptides e)fluoroquinolone f)macrolides

(This antibiotic is used to treat infections with gram-negative, aerobic bacteria and acts by irreversibly inhibiting the 30S subunitof the initiation complex) - Aminoglycosides: are bactericidal agents that inhibit bacterial protein synthesis by binding to the 30S ribosomal subunit. Microbial resistance develops when enzymes (e.g., transferases) are produced that inactivate the antibiotic via phosphorylation, acetylation, or adenylation. Aminoglycosides, which are effective against gram-negative bacilli (e.g., E.coli, Klebsiella, Serratia, Proteus, Pseudomonas), can be used instead of fluoroquinolones for the inpatient treatment of complicated pyelonephritis.

A 6-day-old female infant is brought to the emergency department because of poor feeding and irritability for two days. She was born at 39 weeks' gestation, and the pregnancy and delivery were uncomplicated. Her temperature is 39.2°C (102.6°F). She appears lethargic and makes occasional twitching movements in both upper extremities. The anterior fontanelle is soft and full. A lumbar puncture is performed and analysis of the cerebrospinal fluid shows increased protein and decreased glucose. Cerebrospinal fluid culture shows gram-positive, intracellular rods with tumbling motility. Infection of a healthy adult with the pathogen affecting this infant would most likely present with which of the following clinical conditions? a)Cystitis b)Pneumonia c)Meningitis d)Gastroenteritis

(This baby has signs and symptoms of meningitis. Gram-positive intracellular rods with tumbling motility is suggestive of infection with Listeria monocytogenes.) - Gastroenteritis:Listeria monocytogenes most commonly causes self-limited gastroenteritis in healthy adults. It classically is accompanied by a history of dairy product or deli meat consumption (it grows well at refrigeration temperatures). Although typically a self-limiting condition in healthy adults, Listeria infection is more concerning in infants, the elderly, and those with compromised immune systems. These high-risk individuals should be treated with ampicillin.

A 5-year-old boy is brought to the physician because of a painful, burning rash on his left arm for 3 days. Three years ago, he was diagnosed with heart failure due to congenital heart disease and received an allogeneic heart transplantation. He takes cyclosporine to prevent chronic transplant rejection. He has not received any routine childhood vaccinations. A photograph of the rash is shown. Microscopic examination of a skin biopsy specimen is most likely to show which of the following findings? a)Eosinophilic spongiosis and subepidermal blister formation b)Multinucleated epidermal giant cells and intranuclear inclusions c)Papillary microabscesses and granular deposits of IgA d)Fungal hyphae and hyperkeratosis e)Gram-positive cocci and spongiotic dermatitis

(This boy has a painful rash of grouped vesicles on an erythematous base in a dermatomal distribution (C7-C8), which is characteristic of shingles) - Multinucleated epidermal giant cells and intranuclear inclusions: Multinucleated giant cells and Cowdry A inclusions are characteristic histopathologic findings found on a Tzanck test of shingles lesions. Initial infection with varicella zoster virus (VZV) causes chickenpox, which this patient likely developed because he has not received any vaccinations. Immunosuppression(e.g., from cyclosporine taken to prevent chronic transplant rejection) is a risk factor for reactivation of VZV (shingles) after a period of latency in dorsal root ganglia. Other risk factors include older age, HIVinfection, and malnutrition.

A 6-year-old boy is brought to the emergency department because of worsening confusion for the last hour. He has had high-grade fever, productive cough, fatigue, and malaise for the past 2 days. He has not seen a physician in several years. His temperature is 38.9°C (102°F), pulse is 133/min, respirations are 33/min, and blood pressure is 86/48 mm Hg. He is lethargic and minimally responsive. Mucous membranes are dry. Pulmonary examination shows subcostal retractions and coarse crackles bilaterally. Laboratory studies show a hemoglobin concentration of 8.4 g/dL and a leukocyte count of 16,000/mm3. A peripheral blood smear shows sickled red blood cells. Which of the following pathogens is the most likely cause of this patient's current condition? a)Salmonella paratyphi b)Streptococcus pneumoniae c)Neisseria meningitidis d)Staphylococcus aureus

(This boy, who has sickle cell disease, has developed altered mental status, fever, hypotension, and leukocytosis; these features are concerning for a diagnosis of sepsis. The sepsis in this patient is most likely secondary to pneumonia, which caused his productive cough, tachypnea, and coarse crackles. Patients with sickle cell disease have an increased risk of infection by encapsulated organisms.) - Streptococcus pneumoniae: The spleen is important for the opsonization and elimination of encapsulated bacteria. Patients with sickle cell disease often develop functional asplenia by the age of 2-4 years due to repeated episodes of splenic infarction (autosplenectomy) and are therefore at increased risk for infections by encapsulated pathogens such as Streptococcus pneumoniae, Neisseria meningitidis, Haemophilus influenzae type b and Salmonella species. Of all the encapsulated pathogens, Streptococcus pneumoniae is the most common cause of community-acquired bacterial pneumonia and sepsis in patients with sickle cell disease.

A 5-year-old boy is brought to the physician because of a nonpruritic rash on his face that began 5 days ago. It started as a bug bite on his chin that then developed into small pustules with surrounding redness. He has not yet received any routine childhood vaccinations. Physical examination shows small, clustered lesions with gold crusts along the lower lip and chin and submandibular lymphadenopathy. At a follow-up examination 2 weeks later, his serum anti-deoxyribonuclease B antibody titer is elevated. This patient is at greatest risk for which of the following complications? a)Rheumatic fever b)Reactive arthritis c)Shingles d)Glomerulonephritis

(This child's gold-crusted facial lesions are consistent with impetigo, a cutaneous infection that may be caused by Staphylococcus aureus, Streptococcus pyogenes, or coinfection with both pathogens. The elevated anti-deoxyribonuclease B (anti-DNase B) antibody titer indicates that the causal pathogen was S. pyogenes.) - Acute post-streptococcal glomerulonephritis (PSGN) is a common complication of impetigo and may occur in response to an infection with nephritogenic strains of group A streptococci. It is caused by immune complex deposition within the glomerular basement membrane, which triggers complement activation and subsequent destruction of the glomeruli (type 3 hypersensitivity reaction). PSGNtypically arises 10-30 days following infection, with symptoms such as hematuria, edema, and hypertension.

A 73-year-old man is brought to the emergency department because of fever, headaches, and confusion for the past 24 hours. Three years ago, he underwent heart transplantation because of congestive heart failure. His temperature is 38.1°C (100.5°F). He is oriented only to person. Physical examination shows nuchal rigidity. A cerebrospinal fluid culture on blood agar grows colonies of a gram-positive bacillus surrounded by a narrow transparent rim. Administration of which of the following antibiotics is most likely to be effective in the treatment of this patient's condition? a)Chloramphenicol b)Ampicillin c)Ceftriaxone d)Vancomycin

(This elderly patient presents with fever, headaches, altered mental status, immunosuppression (heart transplant recipient), and CSF culture showing a gram-positive rod with a narrow zone of beta-hemolysis. These signs and symptoms are suggestive of meningitis due to Listeria monocytogenes.) - Ampicillin is the most effective drugs for listerial CNS infection and, therefore, the treatment of choice for affected patients. In immunocompromised patients, neonates, and patients with Listeria infection affecting the endocardium or the central nervous system, gentamicin is added in empiric therapy for its synergistic effect.

A previously healthy 17-year-old female college student comes to the emergency department because of a 1-dayhistory of fever, chills, and severe headache. Her temperature is 39.4°C (103°F). Physical examination shows nuchal rigidity and photophobia. Blood cultures are obtained and a lumbar puncture is performed. A Gram stain of cerebrospinal fluid shows gram-negative cocci in pairs. Which of the following virulence factors was most likely involved in the initial step of this patient's infection? a)Alpha toxin b)Protein A c)Lipooligosaccharides d)Adhesive pili e)Polysaccharide capsule

(This girl presents with symptoms of meningitis (i.e., fever, headache, photophobia, nuchal rigidity). The presence of gram-negative diplococci on CSF analysis indicates that the causal organism is Neisseria meningitidis.) - Pili and certain proteins, e.g., opacity proteins, are important virulence factors for N. meningitidis as they allow the bacterium to adhere to and invade epithelial and endothelial cells. Colonization of the nasal epithelium is considered the first crucial step leading to clinically apparent infection. In a small proportion of individuals colonized with N. meningitidis, the bacteria invade the bloodstream and gain access to the brain, presumably via the brain postcapillary venules or the choroid plexus, to cause meningeal infection.

A 6-year-old girl is brought to the physician by her father because of a 3-day history of sore throat, abdominal pain, nausea, vomiting, and high fever. She has been taking acetaminophen for the fever. Physical examination shows cervical lymphadenopathy, pharyngeal erythema, and a bright red tongue. Examination of the skin shows a generalized erythematous rash with a rough surface that spares the area around the mouth. Which of the following is the most likely underlying mechanism of this patient's rash? a)Drug-induced type IV hypersensitivity reaction b) Anti-M protein antibody cross-reaction c)Subepithelial immune complex deposition d) Erythrogenic toxin-induced cytokine release e)

(This girl presents with typical clinical findings of scarlet fever including high fever, cervical lymphadenopathy, a fine sandpaper-like rash with circumoral pallor, tonsillopharyngitis, and strawberry tongue.) - Erythrogenic toxin-induced cytokine release: This patient has typical features of scarlet fever due to Streptococcus pyogenes (group A streptococcus, GAS), a gram-positive bacteria whose release of erythrogenic toxin A, B, or C, results in excessive release of inflammatory mediators. The rash in scarlet fever blanches with pressure and is most pronounced in flexural areas such as axillae, popliteal fossae, and the inguinal region. Other toxigenic effects of GAS infection include streptococcal toxic shock syndrome, erysipelas, and necrotizing fasciitis.

A 37-year-old man comes to the physician because of fever, night sweats, malaise, dyspnea, and a productive cough with bloody sputum for 4 days. He was diagnosed with HIV infection 15 years ago and has not been compliant with his medication regimen. Physical examination shows diminished breath sounds over the left lung fields. An x-ray of the chest shows an ill-defined lesion in the upper lobe of the left lung. A CT-guided biopsy of the lesion is performed; a photomicrograph of the biopsy specimen stained with mucicarmine is shown. Which of the following is the most likely causal organism? a)Cocciodioides immitis b)Candida albicans c)Pneumocystis jirovecii d)Histoplasma capsulatum e)Blastomyces dermatitidis f)Cryptococcus neoformans g)Aspergillus fumigatus

(This immunocompromised (HIV) patient has pneumonia (fever, night sweats, malaise, dyspnea, a productive cough with bloody sputum, diminished breath sounds, and lesions on x-ray). The photograph shows ovoid yeast and some narrow-necked budding. Mucicarmine is a narrow-spectrum stain used to identify one particular fungal organism with a thick polysaccharide capsule) - Cryptococcus neoformans is a pathogenic fungus that has a thick polysaccharide capsule and, therefore, stains bright red with mucicarmine. The fungus is primarily transmitted via soil or pigeon droppings and initially infects the respiratory tract. Infection is typically asymptomatic, but cryptococcosis may occur, especially in immunodeficient individuals (e.g., with AIDS). Cryptococcosis usually manifests as isolated pneumonia, as seen in this patient, or as cryptococcal meningoencephalitis.

A 33-year-old man with HIV comes to the physician because of a nonproductive cough and shortness of breath for 3 weeks. He feels tired after walking up a flight of stairs and after long conversations on the phone. He appears chronically ill. His temperature is 38.5°C (101.3°F), and pulse is 110/min. Pulse oximetry on room air shows an oxygen saturation of 95%. Upon walking, his oxygen saturation decreases to 85%. Cardiopulmonary examination is normal. Laboratory studies show a CD4+ T-lymphocyte count of 176/mm3. Results of urine Legionella antigen testing are negative. A CT scan of the chest shows diffuse, bilateral ground-glass opacities. Microscopic examination of fluid obtained from bronchoalveolar lavage will most likely show which of the following findings? a)Gram-positive, catalase-positive cocci b)Silver-staining, disc-shaped cysts c)Intracellular, acid-fast bacteria d)Septate, acute-branching hyphae

(This immunosuppressed patient presents with indolent progression of nonproductive cough, dyspnea on exertion, fever, a normal lung examination, and bilateral ground-glass opacities on chest CT scan, all of which suggest a diagnosis of Pneumocystis jirovecii pneumonia.) - Silver-staining, disc-shaped cysts on microscopic examination of bronchoalveolar lavage fluid indicate Pneumocystis jirovecii, a yeast-like fungus that causes opportunistic infection in immunocompromisedpatients, particularly those with HIV/AIDS and a CD4+ count below 200 cells/mm3. Pneumocystispneumonia generally manifests with an indolent progression of respiratory symptoms, normal lung examination (especially early in the course of the disease), low oxygen saturation that worsens with exertion, and bilateral, diffuse, interstitial infiltrates (ground-glass opacities) on radiographic imaging of the chest, as seen in this patient.

A 55-year-old man comes to the physician because of fever, fatigue, dry cough, headache, and myalgia over the past week. Two days ago, he developed several painful oral lesions and difficulty swallowing. He underwent kidney transplantation 3 years ago. His temperature is 38.2°C (100.7°F). Physical examination shows bilateral rales, hepatosplenomegaly, and multiple 1-2 cm ulcerative lesions with raised borders in the oral mucosa. A photomicrograph of a liver biopsy specimen is shown. Which of the following is the most likely causal pathogen? a)Aspergillus fumigatus b)Blastomyces dermatitidis c)Histoplasma capsulatum d)Cryptococcus neoformans e)Paracoccidioides brasiliensis f)Mucor species e)Coccidioides immitis

(This infection is most common in the Mississippi and Ohio river valley and occurs from exposure to bird or bat excrement.) - Histoplasma capsulatum: Disseminated Histoplasma capsulatum, which occurs in immunocompromised patients, is the cause of this patient's acute onset fever, fatigue, dry cough, headache, myalgias, hepatosplenomegaly, bilateral rales, and ulcerative oral lesions. The presence of ulcerative oral lesions is particular to this organism in relation to other fungal organisms. The liver biopsy shows the typical findings: macrophages filled with phagocytosed Histoplasma organisms.

A 32-year-old man is brought to the physician by his wife for a 3-day history of fever, headaches, and myalgias. He returned from a camping trip in Oklahoma 10 days ago. He works as a computer salesman. His temperature is 38.1°C (100.6°F). Neurologic examination shows a sustained clonus of the right ankle following sudden passive dorsiflexion. He is disoriented to place and time but recognizes his wife. Laboratory studies show a leukocyte count of 1,700/mm3 and a platelet count of 46,000/mm3. A peripheral blood smear shows monocytes with intracytoplasmic morulae. Which of the following is the most likely causal organism? a)Babesia microti b)Rickettsia rickettsii c)Anaplasma phagocytophilum d)Ehrlichia chaffeensis

(This infection is typically transmitted by Amblyomma americanum (lone star tick). - Ehrlichia spp., such as Ehrlichia chaffeensis, can cause ehrlichiosis, a tick-borne infection that is endemicto the southeastern and south central regions of the US (e.g., Oklahoma). Ehrlichiosis typically manifests with flu-like symptoms (e.g., fever, headaches, and myalgias) and neurologic symptoms (e.g., confusion, clonus). Laboratory findings include leukopenia, thrombocytopenia, elevated serum transaminases, lactate dehydrogenase, and alkaline phosphatase. Unlike other tick-borne infections, ehrlichiosis does not typically cause a rash. The drug of choice for treatment is doxycycline.

A 27-year-old man with a history of intravenous drug use comes to the physician because of anorexia, nausea, dark urine, and abdominal pain for 2 weeks. Physical examination shows scleral icterus and right upper quadrant tenderness. Serum studies show: ALT 1248 AST 980 anti hep B surface antibody + Further evaluation shows hepatitis C virus RNA detected by PCR. Without appropriate treatment, which of the following is the most likely outcome of this patient's current condition? a)Hepatocellular carcinoma b)Slowly progressive hepatitis c)Liver cirrhosis d)Transient infection e)Fulminant hepatitis

(This known IV drug user has symptoms of acute hepatitis. The presence of HCV RNA in combination with elevated transaminases and negative anti-hepatitis C antibodies is indicative of acute hepatitis C (HCV) infection.) - Slowly progressive hepatitis is the most likely natural disease course of acute HCV infection. Between 50 and 85% of patients develop chronic hepatitis. Both viral and individual host factors (e.g., genetic polymorphisms) contribute to the overall risk of viral persistence. Up to one-third of chronically infected individuals develop liver cirrhosis within 20 years. As chronic HCV infection and even liver cirrhosis are frequently asymptomatic or present with nonspecific symptoms until late stages, diagnosis and treatment are often delayed.

A 45-year-old man comes to the emergency department with fever, nonproductive cough, and difficulty breathing. Three years ago, he underwent lung transplantation. A CT scan of the chest shows diffuse bilateral ground-glassopacities. Pathologic examination of a transbronchial lung biopsy specimen shows several large cells containing intranuclear inclusions with a clear halo. Treatment with ganciclovir fails to improve his symptoms. He is subsequently treated successfully with another medication. This drug does not require activation by viral kinases and also has known in-vitro activity against HIV and HBV. The patient was most likely treated with which of the following drugs? a)Foscarnet b)Lamivudine c)Elvitegravir d)Acyclovir

(This lung transplant recipient has CMV pneumonitis (ground-glass opacities, large cells with "owl eye" inclusions) resistant to ganciclovir. The drug that was used to treat this patient suppresses the action of both viral DNA and RNA polymerase as well as HIV reverse transcriptase by inhibiting the pyrophosphate binding sites of the enzymes.) - Foscarnet directly inhibits viral DNA and RNA polymerase (i.e., does not require activation by viral kinase). It is used to treat CMV infections in immunocompromised patients when ganciclovir fails, as in this patient, and acyclovir-resistant HSV infections. Additionally, foscarnet can inhibit viral reverse transcriptase and has an in-vitro activity against HIV and HBV. Significant side effects include nephrotoxicity and serious electrolyte anomalies.

A 68-year-old man is brought to the emergency department 30 minutes after the onset of uncontrollable jerking movements of his arms and legs followed by loss of consciousness. His wife says that he seemed confused this morning and had a headache. Immediately before the shaking episode, he said that he smelled rotten eggs. He is unresponsive. Cerebrospinal fluid (CSF) analysis shows a leukocyte count of 700/μL (70% lymphocytes), a glucose concentration of 60 mg/dL, and a protein concentration of 80 mg/dL. Despite appropriate lifesaving measures, the man dies. Which of the following is most likely to be found on postmortem examination of this patient? a)Hemorrhage into the adrenal glands b)Necrosis of the temporal lobes c)Cytoplasmic inclusions in cerebellar Purkinje cells d)Atrophy of the mammillary bodies

(This man had viral encephalitis, which manifests with altered mental status, seizures and lymphocytic pleocytosis with normal glucose concentration and mildly elevated protein concentration on CSF analysis. - Necrosis of the temporal lobes is a relatively specific finding in herpes simplex encephalitis, which is most commonly caused by HSV-1. Damage to the temporal lobe can cause clinical features such as olfactory hallucinations and aphasia. Biopsy of the lesion would show intranuclear eosinophilic inclusions, which is characteristic of herpes virus infections. HSV encephalitis is fatal in up to 70% of cases if left untreated; intravenous acyclovir is the first-line therapy.

A 61-year-old man is brought to the emergency department by his wife because of increasing confusion over the past 12 hours. His wife reports that he has a history of type 1 diabetes mellitus. His temperature is 38.8°C (101.8°F). He is confused and oriented only to person. Examination shows left periorbital swelling that is tender to palpation, mucopurulent rhinorrhea, and a black necrotic spot over the nose. There is discharge of the left eye with associated proptosis. A photomicrograph of a specimen obtained on biopsy of the left maxillary sinus is shown. Which of the following is the most likely causal organism? a)Candida albicans b)Haemophilus influenzae c)Pseudomonas aeruginosa d)Rhizopus microsporus e)Streptococcus pneumoniae f)Aspergillus fumigatus g)Blastomyces dermatitidis

(This man has a rhino-orbital-cerebral infection caused by a pathogen that is most commonly seen in patients with diabetic ketoacidosis.) - Rhizopus and Mucor species have broad, nonseptate hyphae that branch at right angles, as seen in the photomicrograph, and can cause mucormycosis. Diabetic ketoacidosis is a significant risk factor that predisposes to this condition, which typically manifests as a unilateral, invasive, maxillofacial infection with headache, facial pain, nasal discharge, and fever, as seen in this patient. Severe infections can lead to altered mental status (due to cerebral involvement), eschars (due to microvascular occlusion and resulting tissue necrosis), and cranial nerve palsies.

A 24-year-old man is brought to the emergency department by his brother because of a 3-hour history of lethargy and confusion. The brother says that 2 days ago, the patient ate several large-capped mushrooms he had foraged in the woods. After eating the mushrooms, he developed severe, bloody diarrhea that has since resolved. His pulse is 140/min, respirations are 26/min, and blood pressure is 98/62 mm Hg. Examination shows dry mucous membranes and tenderness to deep palpation in the right upper quadrant. Serum studies show a serum AST concentration of 2335 U/L and ALT concentration of 2294 U/L. Inhibition of which of the following processes is the most likely cause of this patient's condition? a) Messenger RNA synthesis b) ATP production c) Microtubule polymerization d)Parasympathetic activation

(This man has fulminant liver failure, as evidenced by his right upper quadrant pain, highly elevated serum aminotransferaselevels, and acute-onset confusion and lethargy (most likely caused by hepatic encephalopathy). In combination with preceding diarrhea and consumption of large-capped wild mushrooms, this suggests a diagnosis of Amanita phalloides poisoning) -Messenger RNA synthesis: Messenger RNA synthesis is inhibited by α-amanitin, a toxin produced by Amanita phalloides. This toxin is preferentially taken up by hepatocytes and inhibits RNA polymerase II, which results in severe hepatotoxicity. Amanita phalloides poisoning classically manifests with severe diarrhea 6-24 hours after ingestion that resolves 24-36 hours after ingestion. Fulminant hepatic and multiorgan failure occur 2-4 days after ingestion

A 62-year-old man comes to the physician because of a 2-day history of fever, chills, and flank pain. Five days ago, he was catheterized for acute urinary retention. His temperature is 39.3°C (102.7°F). Physical examination shows right-sided costovertebral angle tenderness. Urine studies show numerous bacteria and WBC casts. Urine culture on blood agar grows mucoid, gray-white colonies. Urine culture on eosin methylene blue agar grows purple colonies with no metallic green sheen. Which of the following is the most likely causal pathogen? a)Proteus mirabilis b)Pseudomonas aeruginosa c)Enterococcus faecalis d)Klebsiella pneumoniae

(This man has nosocomial acute pyelonephritis after recent urinary catheterization. The mucoid appearance of the colonies is due to the presence of a thick polysaccharide capsule. The growth pattern on eosin methylene blue agar indicates that the causal pathogen is a lactose fermenter.) - Klebsiella is a gram-negative, lactose-fermenting, rod-shaped bacillus with a particularly thick polysaccharide capsule that results in the growth of mucoid colonies. Like other lactose fermenters, Klebsiella spp. cause eosin methylene blue agar to change in color to purple because they render the surrounding medium acidic. Urinary tract infections are the most common nosocomial infection caused by Klebsiella spp., as seen in this case. The majority of these infections are caused by Klebsiella pneumoniae. Treatment is typically antibiotic therapy with a fluoroquinolone or 3rd-generation cephalosporin, though the incidence of infections by extended-spectrum beta-lactamase-producing bacteria is increasing.

A 44-year-old man comes to the physician for a follow-up examination. Ten months ago, he was diagnosed with HIV infection and appropriate antiretroviral therapy was initiated. Physical examination shows no abnormalities. Laboratory studies show increased viral load despite ongoing treatment. His pharmacotherapy is switched to a new combination drug regimen including an agent that binds to glycoprotein 41. The expected effect of this drug is most likely due to inhibition of which of the following? a)Viral particle assembly b)Viral docking and attachment to host cells c)Viral genome transcription d)Viral genome integration into host cells e)Viral fusion and entry into host cells

(This medication can only be administered parenterally. It is known to cause localized skin reactions at the site of injection) -Viral fusion and entry into host cells: HIV fusion and entry into the host cell requires docking of the viral gp120 with CD4 receptors and a co-receptor. This is followed by a conformational change of gp41 that allows fusion of the viral and cellular membranes. Fusion inhibitors such as enfuvirtide are peptides that bind to gp41, disrupting its function and thereby preventing viral entry into the host cells. Enfuvirtide is reserved as a salvage therapy in cases of resistance. The other side effects of enfuvirtide besides injection site reactions are fatigue and diarrhea.

A 32-year-old man comes to the physician because of a 3-week history of cough, weight loss, and night sweats. He migrated from Sri Lanka 6 months ago. He appears emaciated. His temperature is 38.1°C (100.5°F). Physical examination shows enlargement of the right supraclavicular lymph node. Chest and abdominal examination show no abnormalities. An interferon-gamma assay is positive. A biopsy specimen of the cervical lymph node is most likely to show the causal organism in which of the following locations? a)Germinal center b)Medullary sinus c)Paracortex d)Mantle zone e)Periarteriolar lymphatic sheath

(This migrant patient with weight loss, night sweats, a cough, and a positive interferon gamma assay most likely has active pulmonary tuberculosis, which is typically caused by Mycobacterium tuberculosis. M. tuberculosis, which is primarily ingested by macrophages, has sulfatides that prevent phagolysosomal fusion and can, therefore, persist in macrophages.) - The medullary sinuses are vessel-like spaces between the cords of lymphatic tissue that are located in the medulla of a lymph node. The medullary sinus is filled with macrophages and reticular cells. Histiocytes, a form of macrophage in the medulla, span a network of pseudopods that filter the sinuses of incoming pathogens. M. tuberculosis can, therefore, be seen within macrophages in the medullary sinus.

A 5-day-old boy is brought to the emergency department because of a 1-day history of poor feeding, irritability, and noisy breathing. The mother did not receive any prenatal care. His respirations are 26/min. Physical examination shows sunken fontanelles, tightly clenched fists, and erythema around the umbilical cord stump. Which of the following best describes the pathogenesis of the disease process in this patient? a)Decreased release of acetylcholine b)Destruction of Schwann cells c)Decreased release of glycine d)Inhibition of voltage-gated sodium channels

(This newborn has several risk factors (lack of maternal prenatal care, umbilical cord stump infection) that should raise concern for neonatal tetanus.) - Decreased release of glycine: Neonatal tetanus is caused by tetanospasmin, which is produced by ubiquitous Clostridium tetani spores that then contaminate wounds (e.g., umbilical cord). The toxin reaches the CNS via retrograde axonal transport and blocks the release of glycine and GABA from inhibitory interneurons (Renshaw cells). This causes uncontrolled activation of alpha motor neurons, which results in tonic and clonic spasms (difficulty opening the mouth and feeding due to trismus, clenched hands).

A 2980-g (6.6-lb) female newborn is brought to the emergency department by her mother because of worsening lethargy. The newborn was delivered at home 10 hours ago. The mother has had no prenatal care. The newborn's temperature is 39.7°C (103.5°F). Physical examination shows scleral icterus. Her leukocyte count is 36,000/mm3 (85% segmented neutrophils). An organism is isolated from the blood. When grown together on sheep agar, the isolated organism enlarges the area of clear hemolysis formed by Staphylococcus aureus. Which of the following is the most likely causal organism? a)Clostridium botulinum b)Pseudomonas aeruginosa c)Listeria monocytogenes d)Listeria monocytogenes e)Streptocccus pneumoniae f)Streptococcus agalactiae

(This newborn's fever, lethargy, and elevated leukocyte count are concerning for neonatal sepsis, which is often accompanied by jaundice and dehydration. The causal organism produces CAMP factor, a protein that enhances the effect of β-lysin of S. aureus, thereby enlarging the area of clear hemolysis.) - Streptococcus agalactiae (i.e., group B streptococcus (GBS)) are β-hemolytic, gram-positive cocci that colonize the vaginal mucosa and are among the most common pathogens of both early- and late-onset neonatal sepsis. Pregnant women are routinely screened at 35-37 weeks of gestation with rectal and vaginal swabs. Transmission of GBS to neonates can be prevented via administration of intrapartum penicillin prophylaxis. This patient's lack of prenatal care increased her child's risk of GBS sepsis.

An investigator is studying the antimicrobial resistance profile of a bacterial pathogen using disk diffusion testing. The pathogen is isolated onto agar plates and antibiotic wafers are added to the plates. The inhibitory zone diameters are then recorded and used to define susceptibility thresholds. The results of the testing for the pathogen shown: penicillin g, cefazoline, imipenem, vancomycin: resistant doxycycline, azythromycin: susceptible which of the following is the most likely organism studied? a) Staphylococcus aureus b)Clostridioides difficile c)Treponema pallidum d)Pseudomonas aeruginosa e)Ureaplasma urealyticum

(This pathogen shows resistance to antibiotics that affect the bacterial cell wall, which indicates that this organism lacks a cell wall) - Ureaplasma urealyticum: Ureaplasma urealyticum, a bacterium closely related to Mycoplasma spp., is the most likely organism. Unlike many other species of bacteria, Ureaplasma do not have cell walls. As beta-lactam antibioticssuch as penicillins, cephalosporins, and carbapenems act by inhibiting the synthesis of peptidoglycan, a bacterial cell wall protein, these drugs are ineffective against Ureaplasma. Ureaplasma spp. are most commonly associated with urinary tract infections and pelvic inflammatory disease, and more rarely with more severe diseases such as pneumonia and meningitis. Both tetracyclines (doxycycline) and macrolides (e.g., azithromycin) are drugs of choice for treatment

A 44-year-old man with HIV comes to the physician for a routine follow-up examination. He has been noncompliant with his antiretroviral medication regimen for several years. He appears chronically ill and fatigued. CD4+ T-lymphocyte count is 405/mm3. Further evaluation of this patient is most likely to show which of the following findings? a)Violaceous lesions on skin exam b)Cavitations on chest X-ray c)White plaques on esophagogastroduodenoscopy d)Cotton-wool spots on fundoscopy e)Encapsulated yeast on India ink staining of blood f)Multifocal demyelination on brain MRI g)Ring-enhancing lesions on brain MRI h)Ground-glass opacities on chest CT

(This patient has a CD4 T-cell count below 500/mm3, but greater than 200/mm3, putting him at risk for several opportunistic infections, including oral hairy leukoplakia, oral thrush, HHV-8 infection, and squamous cell carcinoma.) - Violaceous lesions on skin exam: Violaceous skin lesions in a patient with HIV describe Kaposi sarcoma. HIV patients are at a higher risk of Kaposi sarcoma at variable CD4 counts, even above 400/mm3.

An 84-year-old woman with an indwelling urinary catheter and a history of recurrent nephrolithiasis is brought to the emergency department from her nursing home because of increasing confusion over the past day. On arrival, she is oriented only to person. Her temperature is 38.3°C (100.9°F). Examination shows dry mucous membranes. Urine studies show: Ph 8.3 nitrites + The urine has an ammonia odor. Which of the following is most likely to be present on this patient's urine culture? a)Gram-negative, oxidase-positive rods b)Gram-positive, novobiocin-resistant cocci c)Gram-negative, lactose-fermenting rods d)Gram-positive, gamma-hemolytic cocci e)Gram-negative, oxidase-negative rods

(This patient has a catheter-associated urinary tract infection (positive nitrites and moderate bacteria on urinalysis) with resulting delirium and sepsis. The organism most likely responsible produces urease, which can convert urea into ammonia, causing elevated urinary pH. Ammonia lends the urine a characteristic scent) - Gram-negative, oxidase-negative rods: Patients with a urinary tract infection (UTI), alkaline urine (pH > 7), and an indwelling urinary catheter are most likely infected with Proteus mirabilis, a gram-negative, oxidase-negative rod. P. mirabilis converts urea in the urine into ammonia, which has an alkalizing effect and increases the risk of magnesium ammonium phosphate stone (e.g., struvite stone) formation. The incidence of P. mirabilis increases with the duration of catheterization and can be found in up to 40% of patients with long-term catheterization.

A 70-year-old man is brought to the emergency department with painful discharge from his right ear with difficulty hearing for 3 days. His temperature is 39.5°C (103.1°F) and pulse is 120/minute. Physical examination shows mild facial asymmetry with the right corner of his mouth lagging behind the left when the patient smiles. There is severe ear pain when the right auricle is pulled superiorly. Otoscopic examination shows granulation tissue at the transition between the cartilaginous and the osseous part of the ear canal. Which of the following is most likely associated with this patient's condition? a)Invasive tumor of external auditory canal b)Decreased intracranial perfusion c)Glucose intolerance d)Positive streptococcal culture

(This patient has a history and physical exam findings consistent with malignant otitis externa given his severe ear pain, vital signabnormalities (i.e., high fever, tachycardia), facial droop (indicating progression to osteomyelitis of the temporal bone with associated CN VII palsy), and the presence of granulation tissue in the ear canal.) - glucoses intolerance: There is a strong association between diabetes mellitus and the development of malignant otitis externa, which is most commonly caused by an infection with Pseudomonas aeruginosa. The specific connection between diabetes mellitus and malignant otitis externa is not fully understood but may be related to increased pH in diabetic cerumen. Elderly diabetic patients are especially at risk for this potentially fatal condition. Following blood cultures, high-dose IV ciprofloxacin should be initiated and cranial imaging performed.

A 62-year-old man comes to the physician because of a growth on his penis that has been gradually increasing in size over the last year. He was diagnosed with HIV 10 years ago. He has been divorced for 25 years and has had "at least 30 sexual partners" since. Physical examination shows a nontender 2.5-cm ulcerated lesion with an erythematous base on the dorsum of the glans. There is firm left inguinal lymphadenopathy. A biopsy of the lesion shows small uniform basophilic cells with central necrosis that invade into the corpus cavernosum. This patient's condition is most likely associated with which of the following pathogens? a)Human papillomavirus b)Epstein-Barr virus c)Haemophilus ducreyi d)Treponema pallidum e)Chlamydia trachomatis

(This patient has advanced carcinoma of the penis!) - HPV, particularly types 16 and 18, is detected in up to 50% of patients with carcinoma of the penis; it most likely has a causative role in carcinogenesis. An increased incidence of carcinoma of the penis is also found in HIV-infected men. Although HIV may directly contribute to carcinogenesis, this association may also be due to the frequent HPV co-infection in this patient group. Apart from the immunosuppression enabling the persistence of HPV infection, HIV also seems to directly promote HPV-associated carcinogenesis.

A 27-year-old woman comes to the physician because of a 3-day history of a sore throat and fever. Her temperature is 38.5°C (101.3°F). Examination shows edematous oropharyngeal mucosa and enlarged tonsils with purulent exudate. There is tender cervical lymphadenopathy. If left untreated, which of the following conditions is most likely to occur in this patient? a)Toxic shock syndrome b)Polymyalgia rheumatica c)Rheumatoid arthritis d)Dilated cardiomyopathy

(This patient has an acute infection of the oropharynx with odynophagia, fever, pharyngeal edema, cervical lymphadenopathy, and tonsillar hypertrophy with exudates. These features suggest group A streptococcal (GAS) pharyngitis.) - Dilated cardiomyopathy is a potential late and serious complication of acute rheumatic fever, a type II hypersensitivity reaction which can occur within 2-4 weeks after untreated group A Streptococcusinfection. Rheumatic heart disease can manifest in several ways, including myocarditis, pericarditis, as well as valvular disease (mitral valve involvement is most common). The development of dilated cardiomyopathy is multifactorial and usually occurs secondary to valvular disease, but can also result directly from myocarditis.

A previously healthy 57-year-old man is brought to the emergency department because of a 3-day history of fever and headache. He also has nausea and vomited twice in the past 24 hours. His temperature is 39.1°C (102.4°F). He is lethargic but oriented to person, place, and time. Examination shows severe neck rigidity with limited active and passive range of motion. A lumbar puncture is performed; cerebrospinal fluid analysis shows a neutrophilic pleocytosis and a decreased glucose concentration. A Gram stain of the patient's cerebrospinal fluid is most likely to show which of the following? a)Non-encapsulated, gram-negative cocci in pairs b)Gram-negative coccobacilli c)Encapsulated, gram-negative cocci in pairs d)Gram-positive cocci in clusters e)Gram-negative bacilli f)Encapsulated, gram-positive cocci in pairs

(This patient has bacterial meningitis, a life-threatening infection that causes a characteristic triad of fever, headache, and neck stiffness. Streptococcus pneumoniae is the most common cause of bacterial meningitis among adults of all ages.) - Encapsulated, gram-positive cocci in pairs: Streptococcus pneumoniae is an encapsulated, gram-positive coccus that arranges in pairs and the most common cause of bacterial meningitis among adults of all ages in the US (> 50% of all reported cases). Initial therapy for S. pneumoniae meningitis includes intravenous vancomycin plus a third-generation cephalosporin (e.g., ceftriaxone, cefotaxime).

An 84-year-old woman is brought by her caretaker to the physician because of a 2-day history of fever, severe headache, neck pain, and aversion to bright light. She appears uncomfortable. Her temperature is 38.5°C (101.3°F), pulse is 110/min, and blood pressure is 145/75 mm Hg. Physical examination shows involuntary flexion of the bilateral hips and knees with passive flexion of the neck. Cerebrospinal fluid analysis shows a leukocyte count of 1200/mm3 (76% segmented neutrophils, 24% lymphocytes), a protein concentration of 113 mg/dL, and a glucose concentration of 21 mg/dL. A CT scan of the brain shows leptomeningeal enhancement. Which of the following is the most appropriate initial pharmacotherapy? a)Vancomycin, gentamicin, and cephalexin b)Vancomycin, metronidazole, and cefotaxime c)Vancomycin and cefepime d)Ampicillin and gentamicin e)Ceftriaxone, vancomycin, and ampicillin

(This patient has classic findings of bacterial meningitis: fever, confusion, Brudzinski sign, and CSF analysis showing ↑ cell count, ↑ protein, and ↓ glucose. The most common causes of community-acquired bacterial meningitis in patients older than 60 yearsare the gram-positive bacteria S. pneumoniae and L. monocytogenes, and the gram-negative bacteria H. influenzae and E. coli.) - Ceftriaxone, vancomycin, and ampicillin: Third generation cephalosporins (e.g., cefotaxime, ceftriaxone) are broad-spectrum antibiotics that are effective against most gram-negative and gram-positive bacteria with the exception of Enterococcus and Listeria. Since L. monocytogenes is an important cause of bacterial meningitis in patients > 60 years, empiric antibiotic therapy in this group should include ampicillin, an anti-listerial antibiotic. Ampicillinwould also provide coverage against gram-positive and gram-negative bacteria, but its spectrum of activity is not as broad as 3rd generation cephalosporins. Vancomycin, which acts only against gram-positive bacteria, should be included in the antibiotic regimen because the incidence of penicllin- and cephalosporin-resistant pneumococcal infection has increased in recent years. Vancomycin, while effective in Listeria bacteremia, is not very effective against CNS Listeria infections. Therefore, ampicillinshould be used despite the addition of vancomycin to the regimen.

A previously healthy 5-year-old boy is brought to the emergency department because of fever, irritability, malaise, and left knee pain for 4 days. Four days ago, he fell off his bike and scraped his elbow. His temperature is 39.1°C (102.4°F). The patient walks with a limp. Examination shows swelling and point tenderness over the medial aspect of the left knee. An MRI of the left knee shows edema of the bone marrow and destruction of the medial metaphysis of the tibia. Which of the following is the most likely causal organism? a)Staphylococcus epidermidis b)Brucella melitensis c)Staphylococcus aureus d)Pseudomonas aeruginosa e)Pasteurella multocida

(This patient has osteomyelitis of the lower extremity. Since there is no evidence of direct inoculation (e.g., trauma to the knee) or contiguous spread (e.g., from a foot ulcer), the most likely route of infection is via hematogenous spread. The most common causal pathogen of hematogenous osteomyelitis in children has several virulence factors, including protein A.) - Staphylococcus aureus accounts for the majority of acute osteomyelitis cases in both children and adults. Hematogenous osteomyelitis occurs more frequently in children than adults; > 50% of patients are 5 years of age or younger. In children, the metaphysis of long bones is most often affected, whereas in adults, the vertebrae are the most common site. Pediatric risk factors include sickle cell disease, immunodeficiency disorders, sepsis, indwelling vascular catheters, and minor trauma.

An 81-year-old man is brought to the emergency department by staff of an assisted living facility where he resides with fever and a cough that produces yellow-green sputum. His temperature is 39.1°C (102.3°F). Physical examination shows diffuse crackles over the right lung fields. An x-ray of the chest shows consolidation in the right lower lobe. Sputum cultures grow an organism that produces blue-green pigments and smells of sweet grapes. Treatment with piperacillin and a second agent is begun. Which of the following is the most likely mechanism of action of the second agent? a)Impairs bacterial degradation of piperacillin b)Inhibits bacterial synthesis of folate c)Prevents the metabolic breakdown of piperacillin d)Inhibits bacterial DNA gyrase

(This patient has pneumonia caused by Pseudomonas aeruginosa, a gram-negative rod that produces pyoverdine and pyocyanin(blue-green pigments) and has a characteristic odor of sweet grapes. This pathogen is treated with a combination of piperacillinand tazobactam.) - Impairs bacterial degradation of piperacillin: Piperacillin, like all penicillins, is a β-lactam antibiotic. P. aeruginosa synthesizes β-lactamase, an enzyme that splits the β-lactam ring, thus inactivating beta-lactam antibiotics. The addition of the β-lactamase inhibitor tazobactam serves two purposes: to reduce the degradation rate of piperacillin and to extend the spectrum of antibiotic activity. For piperacillin to be effective against P. aeruginosa, tazobactamneeds to be added. Other examples of β-lactamase inhibitors include clavulanic acid (combined with amoxicillin) and sulbactam (combined with ampicillin).

A 42-year-old woman with a history of multiple sclerosis and recurrent urinary tract infections comes to the emergency department because of flank pain and fever. Her temperature is 38.8°C (101.8°F). Examination shows left-sided costovertebral angle tenderness. She is admitted to the hospital and started on intravenous vancomycin. Three days later, her symptoms have not improved. Urine culture shows growth of Enterococcus faecalis. Which of the following best describes the most likely mechanism of antibiotic resistance in this patient? a)Production of beta-lactamase b)Alteration of penicillin-binding proteins c)ncreased efflux across bacterial cell membranes d)Alteration of peptidoglycan synthesis

(This patient has pyelonephritis caused by vancomycin-resistant enterococci (VRE). Enterococci can transfer antibiotic resistance to other bacteria via transposition of plasmid-encoded van genes) -Alteration of peptidoglycan synthesis: Vancomycin binds to the d-alanine-d-alanine terminus of peptidoglycan precursors, thereby inhibiting cell wall synthesis of gram-positive bacteria (e.g., Enterococci). Acquisition of van genes (e.g., through transposition of plasmid-encoded genes) can result in a change from the d-alanine-d-alanine sequence to a different amino acid sequence, such as d-alanine-d-lactate (vanA and vanB genes), which inhibits binding of vancomycin to peptidoglycan.

A previously healthy 32-year-old woman comes to the emergency department with fatigue and bilateral leg swelling. Her pulse is 92/min, and respirations are 24/min. Physical examination shows jugular venous distention and pitting edema of the lower extremities. Her abdomen is distended with shifting dullness and tender hepatomegaly is present. Cardiovascular examination shows a holosystolic murmur heard best at the left lower sternal border that increases in intensity with inspiration. Which of the following is the most likely predisposing factor for this patient's condition? a)45,XO genotype b)Intravenous drug use c)Vitamin B1 deficiency d)Streptococcal pharyngitis

(This patient has signs of right heart failure (JVD, hepatomegaly, peripheral edema) and a murmur that is consistent with tricuspid regurgitation. TR is best auscultated at the left parasternal region of the 4thICS and manifests as a holosystolic murmur that gets louder on inspiration (Carvallo phenomenon). - Intravenous drug use: Acquired tricuspid valve regurgitation and signs of right-sided heart failure in a previously healthy patient suggest a diagnosis of tricuspid valve endocarditis, for which intravenous drug use (IVDU) is the most likely predisposing factor. The most commonly isolated pathogens in infective endocarditis due to IVDU are Staphylococcus aureus (most common), streptococci, enterococci, and Pseudomonas aeruginosa. Additional causes of tricuspid valve regurgitation in a young patient include rheumatic heart disease, right-sided heart failure, and connective tissue disease (e.g., Marfan syndrome).

A 56-year-old woman is brought to the physician by her husband because of a two-day history of fever, malaise, headaches, and confusion. She recently immigrated from rural China and her immunization status is unknown. Her temperature is 39.1°C (102.4°F). Physical examination shows nuchal rigidity and photophobia. Cerebrospinal fluid analysis shows a neutrophil count of 1,500/mm3. Cerebrospinal fluid cultured on Thayer-Martin agar and normal sheep blood agar shows no growth. However, when the sheep blood agar sample is heated, numerous non-hemolytic, opaque, cream-colored colonies grow. Which of the following characteristics best describes the most likely causal pathogen? a)Gram-negative coccobacillus b)Gram-negative, facultative intracellular bacillus c)Gram-positive, lancet-shaped diplococcus d)Gram-negative diplococcus

(This patient has symptoms and laboratory findings consistent with bacterial meningitis. Chocolate agar is sheep blood that is slowly heated until red blood cells lyse and release factors V (NAD) and X (hematin).) - gram negative coccobacillus: Haemophilus influenzae is a gram-negative coccobacillus that requires factors X (hematin) and V (NAD) for growth. Because this organism is incapable of hemolysis on its own, red blood cells must be lysed by slow heating in order to release these molecules into the growth medium and make them available for bacterial metabolism. Without factors X and V, H. influenzae will not grow. When exposed to these factors, the bacteria can reproduce, which can be seen as the growth of cream-colored colonies. Meningitis caused by this organism is most common in patients who have not received the Hib vaccine.

A 31-year-old man comes to the physician because of a 2-day history of nausea, abdominal discomfort, and yellow discoloration of the eyes. Six weeks ago, he had an episode of fever, joint pain, swollen lymph nodes, and an itchy rash on his trunk and extremities that persisted for 1 to 2 days. He returned from a backpacking trip to Colombia two months ago. His temperature is 39°C (101.8°F). Physical examination shows scleral icterus. Infection with which of the following agents is the most likely cause of this patient's findings? a)Hep B b)Hep A c)Campylobacter d)ETEC

(This patient's current condition (jaundice, nausea) suggest liver pathology and his medical history suggests a serum sickness-likereaction (fever, arthritis, itchy rash). - Hepatitis B: Hepatitis B is associated with a serum sickness-like reaction, which typically occurs during the prodromal period of the infection 1-2 weeks after antigen exposure and is thought to be mediated by immune complex formation. Symptoms of serum sickness-like reaction usually resolve within a few weeks, but symptoms from acute HBV infection may persist for longer, as seen in this patient. Other causes of serum sickness-like reaction include antibiotics (cefaclor, penicillin), streptococcal infection, and vaccines.

A 56-year-old woman is brought to the emergency department because of a 2-day history of fever, chills, nausea, and flank pain. She has multiple sclerosis and a chronic indwelling bladder catheter for neurogenic bladder dysfunction. One week ago, she was discharged from the hospital after treatment for a multiple sclerosis flare. Her temperature is 39.3°C (102.8°C). Physical examination shows marked tenderness in the right costovertebral area. Urine cultures grow a non-lactose fermenting, oxidase-positive, gram-negative rod. The causal organism of this patient's current condition most likely produces which of the following substances? a)Lipoteichoic acid b)K capsule c)Urease d)Toxin B e)Pyoverdine

(This patient most likely has a urinary tract infection (UTI) caused by Pseudomonas aeruginosa. This non-lactose fermenting, oxidase-positive, gram-negative rod is a common cause of nosocomial UTIs, especially in patients with chronic indwelling catheters.) - Pyoverdine is a pigment produced by some species of Pseudomonas, including Pseudomonas aeruginosa, that acts mainly as an iron-chelating agent (siderophore). Together with pyocyanin, pyoverdine serves as an essential virulence factor of this bacterium, ensuring its survival and proliferation in iron-deficient conditions, regulating the production of other virulence factors, and supporting the development of biofilms.

A 46-year-old woman from Ecuador is admitted to the hospital because of tarry-black stools and epigastric pain for 2 weeks. The epigastric pain is relieved after meals, but worsens after 1-2 hours. She has no history of serious illness and takes no medications. Physical examination shows no abnormalities. Fecal occult blood test is positive. Esophagogastroduodenoscopy shows a bleeding duodenal ulcer. Microscopic examination of a duodenal biopsy specimen is most likely to show which of the following? a)Curved, flagellated gram-negative rods b)Gram-negative S-shaped rods with polar flagella c)Irregularly drumstick-shaped gram-positive rods d)Gram-positive lancet-shaped diplococci

(This patient presents with a bleeding duodenal ulcer found on EGD. The most common cause of peptic ulcer disease is infection with H. pylori.) - H. pylori is a curved, flagellated gram-negative rod which mainly colonizes the antrum of the stomach. H. pylori is also catalase positive, oxidase positive, and urease positive.

A 24-year-old man comes to the physician with a 2-day history of fever, crampy abdominal pain, and blood-tingeddiarrhea. He recently returned from a trip to Mexico. His temperature is 38.2°C (100.8°F). Abdominal examination shows diffuse tenderness to palpation; bowel sounds are hyperactive. Stool cultures grow nonlactose fermenting, oxidase-negative, gram-negative rods that do not produce hydrogen sulfide on triple sugar iron agar. Which of the following processes is most likely involved in the pathogenesis of this patient's condition? a)Invasion of colonic microfold cells b)Dissemination via bloodstream c)Inhibition of host cytoskeleton organization d)Overactivation of adenylate cyclase

(This patient presents with a fever, abdominal cramps, and bloody diarrhea, which is concerning for bacterial gastroenteritis. The most likely causal organism is Shigella, a rod-shaped, gram-negative, oxidase-negative, and nonlactose fermenting rod that does not produce H2S on triple sugar iron (TSI) agar.) - Invasion of colonic microfold cells is the mechanism by which Shigella causes host illness. Shigella enters cells via pinocytosis before lysing vesicles and entering the cytoplasm. Once inside the cytoplasm, Shigella produces Shiga toxin and rapidly poisons the cell. This mechanism is key to Shigella's ability to cause disease even in cases of very small exposure to the bacteria (i.e., Shigella has a very low infectiousdose). Once inside the colonic mucosa, Shigella travels from cell to cell via actin polymerization.

A 55-year-old man with type 2 diabetes mellitus comes to the physician because of a 4-day history of fever, chills, nausea, and abdominal pain. He does not use illicit drugs. His temperature is 39°C (102.2°F). Physical examination shows right upper quadrant tenderness. Ultrasonography of the abdomen shows a 6-cm solitary, fluid-filled cavity in the right hepatic lobe. CT-guided percutaneous aspiration of the cavity produces yellowish-green fluid. Culture of the aspirated fluid grows gram-negative, lactose-fermenting rods. Which of the following is the most likely cause of the color of the aspirated fluid? a)Pyoverdine b)Polysaccharide capsule c)Prodigiosin d)Myeloperoxidase

(This patient presents with a pyogenic liver abscess. In adults, E. coli is the most common cause of pyogenic liver abscesses, followed by Klebsiella. Both organisms are gram-negative, lactose-fermenting rods. The yellowish-green fluid is pus, which is predominantly composed of neutrophils.) - Myeloperoxidase (MPO), a lysosomal enzyme produced by neutrophils, is responsible for the yellowish-green color of the pus. Neutrophils store MPO in granules and then release it into the extracellular space where MPO has an antimicrobial effect. MPO has a heme moiety that would cause the color of the pus observed here. A similar color can be seen in Pseudomonas infections in which the greenish pigment pyoverdine is produced by the bacteria. However, pyogenic liver abscesses are rarely caused by Pseudomonas, and MPO is the most likely cause of the yellowish-green coloration.

A 32-year-old man is brought to the emergency department with fever, dyspnea, and impaired consciousness. His wife reports that he has also had an episode of dark urine today. Two weeks ago, he returned from a trip to the Republic of Congo. His temperature is 39.4°C (103°F), pulse is 114/min, and blood pressure is 82/51 mm Hg. Physical examination shows scleral icterus. Decreased breath sounds and expiratory crackles are heard on auscultation of the lungs bilaterally. His hemoglobin concentration is 6.3 g/dL. A blood smear shows red blood cells with normal morphology and ring-shaped inclusions. Further laboratory testing shows normal rates of NADPH production. Which of the following is the most appropriate pharmacotherapy for this patient? a)Artesunate b)Chloroquine c)Primaquine d)atovaquone

(This patient presents with features of severe malaria (shock, jaundice, pulmonary edema, impaired consciousness, and significant anemia), which is most commonly caused by Plasmodium falciparum (falciparum malaria). - Artesunate is an antimalarial drug used for the treatment of severe falciparum malaria. Its mechanisms of action are not fully understood but it most likely generates reactive oxygen species, which cause oxidative stress and thereby damage malarial proteins. It also inhibits EXP1, a membrane glutathione S-transferase, which in turn decreases the amount of glutathione in plasmodia. Every patient should be screened for G6PD deficiency prior to initiation of therapy because artesunate's mechanism of action increases the risk of hemolytic anemia in G6PD deficiency.

A 59-year-old man comes to the physician for the evaluation of generalized fatigue, myalgia, and a pruritic skin rashfor the past 5 months. As a child, he was involved in a motor vehicle accident and required several blood transfusions. Physical examination shows right upper abdominal tenderness, scleral icterus, and well-demarcated, purple, polygonal papules on the wrists bilaterally. Laboratory studies show an elevated replication rate of a hepatotropic virus. Further analysis shows high variability in the genetic sequence that encodes the glycosylated envelope proteins produced by this virus. Which of the following is the most likely explanation for the variability in the genetic sequence of these proteins a)Viral RNA polymerase lacks proofreading ability b)Incorporation of envelope proteins from a second virus c)Integration of viral genes into host cell genome d)Infection with multiple viral genotypes

(This patient presents with signs and symptoms of viral hepatitis. His history of blood transfusions before 1992, the year in which screening for the hepatitis C virus (HCV) in donor blood began, suggests chronic infection with hepatitis C. The pruritic skin rash on the wrists is most likely lichen planus, a common extrahepatic manifestation of chronic infection.) - Hepatitis C virus RNA-dependent RNA polymerase has a very poor proofreading mechanism, leading to frequent errors in viral genes. These errors manifest as variations in the structure of viral proteins. Mutations in the viral glycoprotein envelope allow for antigenic variation and evasion of the host antibody response to these envelope proteins. In addition, HCV replicates very quickly, which enhances the effect of poor proofreading and accelerates the production of antigenically distinct envelopes.

An 8-year-old girl is brought to the emergency room for a 6-hour history of fever, sore throat, and difficulty swallowing. Physical examination shows pooling of oral secretions and inspiratory stridor. Lateral x-ray of the neck shows thickening of the epiglottis and aryepiglottic folds. Throat culture with chocolate agar shows small, gram-negative coccobacilli. The patient's brother is started on the recommended antibiotic for chemoprophylaxis. Which of the following is the primary mechanism of action of this drug? a)Inhibition of DNA-dependent RNA-polymerase b)Inhibition of the 50S ribosomal subunit c)Inhibition of prokaryotic topoisomerase II d)Inhibition of peptidoglycan crosslinking

(This patient presents with the thumbprint sign (thickening of the epiglottis) on x-ray of the neck as well as classic clinical features (dysphagia, drooling, distress) of acute epiglottitis due to Haemophilus influenzae infection. The drug of choice for H. influenzaeprophylaxis is rifampin.) - Inhibition of prokaryotic DNA-dependent RNA polymerase is the mechanism of rifampin, the drug of choice for prophylaxis for Haemophilus influenzae. Side effects of rifampin include hepatotoxicity, development of red/orange bodily fluids, and cytochrome P450 enzyme induction. Rifampin may also be used for prophylaxis of meningococcal meningitis.

A 71-year-old man comes to the emergency department because of pain and swelling in his left leg that started after he cut his foot while swimming in the ocean. He has a history of alcoholic cirrhosis. His temperature is 38.3°C(101.0°F). Examination of the left foot shows a small, purulent wound with surrounding swelling and dusky redness extending to the mid-calf. There are numerous hemorrhagic blisters and the entire lower leg is exquisitely tender to light palpation. There is no crepitus. Blood cultures grow gram-negative bacilli that ferment lactose. Which of the following is the most likely causal organism? a)Shigella flexneri b)Pseudomonas aeruginosa c)Clostridium perfringens d)Streptococcus pyogenes e)Vibrio vulnificus

(This patient presents with typical signs of necrotizing fasciitis (e.g., severe pain, fever, dusky skin, blisters). His recent contact with seawater and history of cirrhosis suggests infection with a certain pathogen.) - Vibrio vulnificus is a gram-negative, lactose-fermenting bacteria found in marine environments that can cause soft tissue infections, as well as gastroenteritis. Patients with underlying liver disease (e.g., cirrhosis, hemochromatosis) are at higher risk of developing severe infection (such as necrotizing fasciitis, as seen in this patient) and have higher mortality rates.

A previously healthy 25-year-old woman comes to the physician because of a 3-week history of fever, recurrent headaches, and anorexia. One month ago, she returned from a camping trip in Chile. Her temperature is 39.3°C(102.8°F). Examination shows a swelling in the left periorbital region and axillary, cervical, and inguinal lymphadenopathy. A blood sample is obtained. Processing of the sample with which of the following methods is most likely to confirm the diagnosis? a)Staining with crystal violet dye b)Staining with silver c)Staining with Giemsa dye d)Staining with India ink

(This patient traveled to an area where Chagas disease is endemic (Central and South America) and now presents with features of infection (e.g., high fever, lymphadenopathy) and unilateral periorbital swelling (Romana sign), which are very characteristic for acute Chagas disease.) - giemsa dye: Chagas disease is caused by the protozoa Trypanosoma cruzi, which is transmitted by inoculation of fecal material of the triatomine reduvid bug into skin or mucous membranes. The reduvid bug preferentially bites on the face ("kissing bug"). The Romana sign occurs when the portal of entry of T. cruzi is the conjunctiva or eyelid. The pathogen subsequently disseminates via the bloodstream. Staining this patient's peripheral blood with stains that contain aniline dyes (e.g, Giemsa stain, Wright stain) would likely reveal the presence of Trypanosoma cruzi trypomastigotes. Treatment with benznidazole or nifurtimox is necessary to prevent complications of Chagas disease such as dilated cardiomyopathy, megaesophagus, and acute megacolon.

Four days after undergoing liver transplantation, a 47-year-old man develops fever, chills, malaise, and confusionwhile in the intensive care unit. His temperature is 39.1°C (102.4°F). Blood cultures grow an organism. Microscopic examination of this organism after incubation at 25°C (77°F) for 3 hours is shown. Which of the following is the most likely causal organism of this patient's symptoms? a) aspergillus b)candida c)cryptococcus d)blastomyces e)mucor f)histoplasma

(This patient who recently underwent liver transplantation is most likely on immunosuppressive therapy. The image shows multiple oval yeast cells with branching, septate hyphal elements. However, the presence of constrictions at the septa indicates that these are not true septate hyphae but rather pseudohyphae formed by the budding of yeast cells.) - Candida albicans: Candida albicans is a dimorphic fungus that appears as budding yeast cells with pseudohyphae at 20-25°C and forms germ tubes at 37°C. The formation of germ tubes can be used to differentiate C. albicans from other species of Candida as well as other yeasts. C. albicans typically causes mucocutaneous infections (e.g., oral thrush) but can result in disseminated hematogenous infection with clinical features of sepsis (e.g., fever, chills, malaise, confusion) in severely immunocompromisedpatients. Treatment of systemic infection consists of fluconazole, echinocandins, or amphotericin B.

A 42-year-old man with AIDS comes to the physician for intermittent fever, nonproductive cough, malaise, decreased appetite, abdominal pain, and a 3.6-kg (8-lb) weight loss over the past month. He has not seen a doctor since he became uninsured 2 years ago. His temperature is 38.3°C (100.9°F). Abdominal examination shows mild, diffuse tenderness throughout the lower quadrants. The liver is palpated 2-3 cm below the right costal margin, and the spleen is palpated 1-2 cm below the left costal margin. His CD4+ T-lymphocyte count is 49/mm3. Blood cultures grow acid-fast organisms. A PPD skin test shows 4 mm of induration. Which of the following is the most appropriate pharmacotherapy for this patient's condition? a) Azithromycin and ethambutol b)Voriconazole c)Rifampin and isoniazid d)Erythromycin

(This patient with AIDS has a very low CD4 cell count (< 50 cells/mm3) and has constitutional symptoms, cough, hepatosplenomegaly, lymphadenopathy, and acid-fast organisms in his blood culture, making disseminated Mycobacterium avium complex (MAC) infection the most likely diagnosis) -Azithromycin and ethambutol: The combination of azithromycin (or clarithromycin) and ethambutol is used to treat disseminated MAC infection, which manifests with constitutional symptoms, hepatosplenomegaly, and lymphadenopathy. In some cases, rifabutin is included as a third drug, particularly for patients with a high mycobacterialload. Patients should also receive antiretroviral HIV therapy to improve the CD4 count.

A 58-year-old woman with HIV infection is brought to the emergency department because of a 2-week history of headaches, blurred vision, and confusion. Her current medications include antiretroviral therapy and trimethoprim-sulfamethoxazole. Neurological examination shows ataxia and memory impairment. Her CD4+ T-lymphocyte count is 90/mm3. Analysis of her cerebrospinal fluid analysis shows lymphocytic predominant pleocytosis, and PCR is positive for Epstein-Barr virus DNA. An MRI of the brain with contrast shows a solitary, weakly ring-enhancing lesion with well-defined borders involving the corpus callosum. Which of the following is the most likely diagnosis? a)AIDS dementia b) Cerebral toxoplasmosis c)Primary cerebral lymphoma d)Progressive multifocal leukoencephalopathy e)Bacterial brain abscess

(This patient with HIV and a low CD4 count presents with neurological impairment (ataxia, confusion, memory dysfunction). The likely diagnosis can be inferred from her MRI findings (a solitary ring-enhancing lesion) and CSF analysis result (EBV DNA in the CSF). -Primary cerebral lymphoma: Primary cerebral lymphoma is a non-Hodgkin lymphoma with a much higher incidence in HIV/AIDSpatients than in the general population. It has a strong association with EBV infection and is derived from B cells. The condition can present with focal neurological deficits, neuropsychiatric symptoms, signs of elevated intracranial pressure, and/or seizures. It also results in solitary ring-enhancing cerebral lesions on imaging. If findings on CSF are inconclusive, pathologic examination of a brain biopsyspecimen is indicated to confirm the diagnosis prior to starting treatment with chemotherapy and/or radiation

An otherwise healthy 56-year-old man comes to the physician for a 2-year history of recurrent upper abdominal pain and fullness that worsens after meals. Urea breath test is positive. An endoscopy shows diffuse mucosal atrophy and patchy erythema, but no ulcer. A biopsy from which of the following areas is most likely to yield an accurate diagnosis? a)Gastric fundus b)Distal esophagus c)Gastric antrum d)Duodenal bulb

(This patient with recurrent upper abdominal pain and fullness after meals (dyspepsia), positive urea breath test, and upper endoscopy findings of diffuse mucosal atrophy and patchy erythema, likely has atrophic gastritis caused by Helicobacter pylori.) - Gastric antrum: Chronic atrophic gastritis due to H. pylori infection begins in the gastric antrum (pyloric antrum) and then spreads to other regions such as the fundus and body. Biopsies from the gastric antrum are therefore most likely to yield an accurate diagnosis because atrophic changes and H. pylori infection load is greatest at this site. Gastric MALToma is a potential complication of H. pylori infection. In contrast to H. pylori-induced atrophic gastritis, the gastric fundus and body are primarily affected in autoimmune atrophic gastritis, which can present similarly but the urea breath test would be negative and other features suggestive of autoimmune disease (e.g., pernicious anemia, vitiligo, autoimmune thyroiditis) would usually be present. Patients with atrophic gastritis have decreased gastric acid secretion (hypochlorhydria) with a secondary increase in gastrin levels.

A 26-year-old man comes to the emergency department for evaluation of burning with urination and purulent urethral discharge for the past 3 days. He is sexually active with multiple female partners. Several months ago he was diagnosed with urethritis caused by gram-negative diplococci and received antibiotic treatment with complete resolution of his symptoms. A Gram stain of the patient's urethral discharge shows gram-negative intracellular diplococci. Which of the following properties of the infecting organism most contributed to the pathogenesis of this patient's recurrent infection? a)Expression of beta-lactamase genes b)Invasion of neutrophilic granulocytes c)Variation of expressed pilus proteins

(This patient's burning sensation with urination and Gram stain results (gram-negative intracellular diplococci) are consistent with gonococcal urethritis. Neisseria gonorrhoeae has developed several virulence factors that prevent host immune recognition.) - Pili are an important virulence factor that mediate the attachment of N. gonorrhoeae to the epithelialcells of a host. Anti-pilin antibodies that are directed against specific pilus proteins could prevent attachment of the pathogen. To circumvent this host immune response, N. gonorrhoeae utilizes DNA-recombination to create a high frequency of antigenic variation of expressed pilus proteins; these proteins thus are no longer recognized by the initial antibodies. This particular virulence factor has made it difficult to create a vaccine that targets the gonococcal pili.

A 53-year-old man comes to the physician because of a 1-day history of fever and chills, severe malaise, and cough with yellow-green sputum. He works as a commercial fisherman on Lake Superior. Current medications include metoprolol and warfarin. His temperature is 38.5°C (101.3°F), pulse is 96/min, respirations are 26/min, and blood pressure is 98/62 mm Hg. Examination shows increased fremitus and bronchial breath sounds over the right middle lung field. An x-ray of the chest shows consolidation of the right upper lobe. Which of the following is the most likely causal pathogen? a)tb b)pseudomonas c)haempohilus d)strep pyogenes e)strep pneumoniae f)legionella

(This patient's symptoms and x-ray findings are diagnostic of community-acquired lobar pneumonia) - Streptococcus pneumoniae: This patient's acute onset of malaise, fever, and chills, productive cough, and tachypnea in combination with the physical examination findings (increased fremitus, bronchial breath sounds) and the chest x-rayshowing lobar consolidation is indicative of typical pneumonia. Streptococcus pneumoniae is the most common causal pathogen

A 62-year-old woman with metastatic breast cancer comes to the physician because of a 2-day history of fever, chills, and new gluteal lesions. The lesions began as painless red macules and evolved into painful ulcers overnight. She received her fourth course of palliative chemotherapy 2 weeks ago. Her temperature is 38.2°C (100.8°F). Laboratory studies show a leukocyte count of 2,000/mm3 (20% segmented neutrophils). A photograph of one of the skin lesions is shown. Which of the following virulence factors is most likely involved in the pathogenesis of this patient's skin finding? a)Edema toxin b)Heat-stable toxin c)Toxic shock syndrome toxin-1 d)Exotoxin A e)Alpha toxin

(This patient's chemotherapy has caused immunosuppression and neutropenia, which increase the risk of developing an infection. She has developed a fever and rapidly evolving gluteal lesions characterized by a circular ulcer with surrounding erythema and a central black eschar. These features are consistent with ecthyma gangrenosum, which is classically associated with P. aeruginosabacteremia.) - Exotoxin A is an AB toxin produced by P. aeruginosa that inhibits protein synthesis by inactivating elongation factor (EF-2). This leads to cell death, which is responsible for the lesions of ecthyma gangrenosum. Other virulence factors produced by P. aeruginosa that also cause cell death include phospholipase C (which degrades cell membranes), elastase (which causes proteolytic damage to tissue, destroying blood vessel walls), and pyocyanin (which creates damaging free radicals and contributes to the blue-green color of some infections). P. aeruginosa also produces endotoxin, which causes fever and shock.

A previously healthy 46-year-old woman comes to the physician with a one-week history of productive cough and fatigue. Two weeks ago, she had fever, nasal congestion, rhinorrhea, and myalgias that resolved with supportive care. She has not traveled out of the United States. Pulmonary examination shows dullness to percussion and increased fremitus at the right middle lobe. An x-ray of the chest is shown. A sputum sample is most likely to show which of the following findings? a) Gram-positive, catalase-positive cocci b)Silver-staining, gram-negative bacilli c)Gram-indeterminate, acid-fast bacteria d)Encapsulated, gram-negative coccobacilli

(This patient's chest x-ray shows a large irregularly rounded cavity with an air-fluid level in the right middle lobe, indicating a lung abscess) -Gram-positive, catalase-positive cocci: A sputum sample is most likely to show Staphylococcus aureus, which can cause lung abscesses, especially in cases of recent or current influenza virus infection (as is likely this case in this patient). Other pathogens implicated in monomicrobial lung abscesses include Klebsiella pneumoniae, and Streptococcus anginosus. Multimicrobial infection with anaerobic bacteria that colonize the oral cavity is more common.

A 60-year-old man comes to the physician because of a 2-month history of cough productive of yellow sputum with occasional streaks of blood. He has a history of pulmonary tuberculosis. He is afebrile. Pulmonary examination shows inspiratory crackles at the left infraclavicular area. An x-ray of his chest shows a radiopaque mass in the left upper lung lobe that shifts upon repositioning. A sputum sample does not grow acid-fast bacteria despite multiple cultures. Which of the following is the most likely cause of this patient's condition? a)Hypersensitivity reaction b)Invasive infection c)Asbestos inhalation d)Neoplastic transformation e)Opportunistic colonization

(This patient's clinical features (productive cough, hemoptysis), history of tuberculosis, and intracavitary mass on chest x-raysuggest chronic pulmonary aspergillosis.) - Opportunistic colonization of a heavily scarred lung cavity (e.g., after a bout of pulmonary tuberculosis) by Aspergillus species describes the pathogenesis of aspergilloma. Aspergilloma (also known as a fungus ball) can manifest with cough, hemoptysis, and a radiopaque mass that shifts upon repositioning, as seen in this patient.

A 53-year-old woman with hypertension is brought to the emergency department 30 minutes after having a generalized, tonic-clonic seizure. She has had recurrent headaches and dizziness in the last 3 months. One year ago, she had diarrhea after a trip to Ecuador that resolved without treatment. She has not received any medical care in the last five years. She has smoked 1 pack of cigarettes daily for 20 years. Her temperature is 36°C (96.8°F) and blood pressure is 159/77mm Hg. Physical examination shows dysarthria and hyperreflexia. She is confused and oriented only to name and place. Four brain lesions are found in a CT scan of the brain; one of the lesions is shown. Which of the following is most likely to have prevented this patient's condition? a) Avoidance of contaminated food b)Vaccination against meningococcus c)Avoidance of cat feces d)Smoking cessation

(This patient's combination of seizure, headache, dizziness, dysarthria, hyperreflexia, confusion, recent travel to South America, and ring-shaped, cystic lesions on brain CT scan indicates a diagnosis of neurocysticercosis) - Avoidance of contaminated food: Avoidance of food contaminated by the eggs of Taenia solium reduces the risk of neurocysticercosis, which is the most common parasitic disease of the nervous system and the most important cause of adult-onset seizures in Asia, Africa, Central and South America. Neurocysticercosis is a fecal-oral transmitted disease that results from ingestion of eggs of Taenia solium, which can be found in food and water contaminated by the feces of people with intestinal taeniasis. Several strategies have been described to prevent the infection, which include washing food prior to consumption, proper hand hygiene, and avoidance of raw pork meat, as well as adequate freezing and cooking of food to destroy the parasite.

A 31-year-old man comes to the physician because of a 2-day history of nausea, abdominal discomfort, and yellow discoloration of the eyes. Six weeks ago, he had an episode of fever, joint pain, swollen lymph nodes, and an itchy rash on his trunk and extremities that persisted for 1 to 2 days. He returned from a backpacking trip to Colombia two months ago. His temperature is 39°C (101.8°F). Physical examination shows scleral icterus. Infection with which of the following agents is the most likely cause of this patient's findings? a)Hepatitis B b)Enterotoxigenic E. coli c)Hepatitis A

(This patient's current condition (jaundice, nausea) suggest liver pathology and his medical history suggests a serum sickness-likereaction (fever, arthritis, itchy rash). - Hepatitis B is associated with a serum sickness-like reaction, which typically occurs during the prodromal period of the infection 1-2 weeks after antigen exposure and is thought to be mediated by immune complex formation. Symptoms of serum sickness-like reaction usually resolve within a few weeks, but symptoms from acute HBV infection may persist for longer, as seen in this patient. Other causes of serum sickness-like reaction include antibiotics (cefaclor, penicillin), streptococcal infection, and vaccines.

A 42-year-old woman with well-controlled HIV on antiretroviral therapy comes to the physician because of a 2-week history of a painless lesion on her right calf. Many years ago, she had a maculopapular rash over her trunk, palms, and soles that resolved spontaneously. Physical examination shows a 4-cm firm, nontender, indurated ulcer with a moist, dark base and rolled edges. There is a similar lesion at the anus. Results of rapid plasma reagin testing are positive. Which of the following findings is most likely on microscopic examination of these lesions? a)Necrotic area surrounded by fibroblast and macrophage infiltrate b)Ulcerated epidermis with plasma cell infiltrate c)Spindle-shaped cells with leukocyte infiltrate and angiogenesis d)Epidermal hyperplasia with dermal lymphocytic infiltrate

(This patient's current skin lesions are most likely gummas, a sign of tertiary syphilis. Her remote history of rash is suggestive of secondary syphilis.) - Necrotic area surrounded by fibroblast and macrophage infiltrate: Gummas are granulomatous skin lesions characteristic of tertiary syphilis. A biopsy of a gumma would most likely show a central area of necrosis (due to obliterating endarteritis, which causes ischemia of the lesion's center) surrounded by an inflammatory cellular infiltrate (due to a type 4 hypersensitivity reaction to Treponemal spirochetes). Gummas may form on the skin, CNS (neurosyphilis), bones, and/or internal organs. They appear in advanced Treponemal pallidum infections and are more common in patients with HIV infection.

A 38-year-old man comes to the physician because of a 2-week history of abdominal pain and an itchy rash on his buttocks. He also has fever, nausea, and diarrhea with mucoid stools. One week ago, the patient returned from Indonesia, where he went for vacation. Physical examination shows erythematous, serpiginous lesions located in the perianal region and the posterior thighs. His leukocyte count is 9,000/mm3 with 25% eosinophils. Further evaluation is most likely to show which of the following findings? a)Rhabditiform larvae on stool microscopy b)Eggs on tape test c)Oocysts on acid-fast stool stain d)Giardia lamblia antibodies on stool immunoassay

(This patient's fever, gastrointestinal symptoms (abdominal pain, nausea, and diarrhea), serpiginous rash, eosinophilia, and recent travel to a tropical area are highly suggestive of strongyloidiasis.) - Detection of rhabditiform larvae on stool microscopy is diagnostic of strongyloidiasis. Strongyloides stercoralis is transmitted by larval penetration of intact skin (usually when bare feet come into contact with contaminated soil). Larvae travel to the alveoli through the bloodstream, where they ascend the pulmonary system to the pharynx before being coughed up and swallowed. Once in the GI tract, they develop into adult female worms and reproduce via parthenogenesis, with the eggs hatching as rhabditiform larvae. Rhabditiform larvae then develop into infective filariform larvae, penetrating intestinal mucosa or the skin of the perianal region, which completes the process of autoinfection. This patient's serpiginous rash and gastrointestinal symptoms are manifestations of the cutaneous and intestinal phases of infection.

A 20-year-old man comes to the physician because of a 3-day history of fever, myalgia, and swelling in his left groin after a recent camping trip in northern California. He appears acutely ill. Physical examination shows tender, left-sided inguinal lymphadenopathy and an enlarged, tender lymph node in the right axilla that is draining bloody necrotic material. Microscopic examination of a lymph node aspirate shows gram-negative coccobacilli with bipolar staining and a safety-pin appearance. This patient's condition is most likely caused by an organism with which of the following reservoirs? a)deer b)squirrels c)birds d)dogs e)bats f)sheep

(This patient's flu-like symptoms, necrotic lymphadenopathy, and history of recent camping in California should raise suspicion for bubonic plague. The lymph node Gram stain is consistent with Yersinia pestis infection) - squirrels: Yersinia pestis, the pathogen responsible for the bubonic plague, is transmitted to humans by fleas and has a natural long-term reservoir in rodents (e.g., squirrels, rats, prairie dogs, rabbits). It is still found in rural areas of the western US (California, Arizona, New Mexico), especially in forests and grasslands. The diagnosis is confirmed via cultures and Wayson staining showing bipolar staining (closed safety-pin appearance), but treatment should begin as soon as bubonic plague is suspected. Aminoglycosides are the treatment of choice for plague

A 43-year-old man hospitalized for acute pancreatitis develops a high-grade fever and productive cough with gelatinous sputum. A sample of his expectorated sputum is obtained and fixed to a microscope slide using heat. A crystal violet dye is applied to the slide, followed by an iodine solution, acetone solution, and lastly, safranin dye. A photomicrograph of the result is shown. Which of the following cell components is responsible for the pink color seen on this stain? a)Peptidoglycan b)Glycogen c)Protein d)Mycolic acid e)Capsular polysaccharide

(This patient's history of high-grade fever and productive cough, in conjunction with pink rods in the Gram stain of his sputum, suggest the nosocomial infection Klebsiella pneumoniae. The pink color of the bacilli comes from the safranin dye in Gram staining.) - Peptidoglycan is the macromolecule in bacterial cell walls that determines a positive or negative Gram stain. K. pneumoniae is a gram-negative bacterium. Gram-negative species have a thin peptidoglycanlayer with an overlying lipid-protein bilayer. In the process of Gram staining, gram-negative species do not retain the applied crystal violet dye after being washed with alcohol, owing to the relatively thin peptidoglycan layer; gram-positive species, due to the thick layer of peptidoglycan in the cell wall, will retain the crystal violet dye and appear deep purple. Instead, gram-negative organisms are visualized in a pink hue after the final addition of safranin as a counterstain.

A 35-year-old man comes to the emergency room for severe left leg pain several hours after injuring himself on a gardening tool. His temperature is 39°C (102.2°F) and his pulse is 105/min. Physical examination of the left leg shows a small laceration on the ankle surrounded by dusky skin and overlying bullae extending to the posterior thigh. There is a crackling sound when the skin is palpated. Surgical exploration shows necrosis of the gastrocnemius muscles and surrounding tissues. Tissue culture shows anaerobic gram-positive rods and a double zone of hemolysis on blood agar. Which of the following best describes the mechanism of cellular damage caused by the responsible pathogen? a)Lipopolysaccharide-induced complement and macrophage activation b)Degradation of cell membranes by phospholipase c)Increase of intracellular cAMP by adenylate cyclase d)Inactivation of 60s ribosome subunit by N-glycosidase

(This patient's myonecrosis is likely caused by alpha toxin produced by Clostridium perfringens.) - Degradation of cell membranes by phospholipase: The alpha toxin produced by Clostridium perfringens is a lecithinase that degrades cell membranephospholipids. Degeneration of membrane phospholipids leads to cell lysis and myonecrosis, resulting in gas gangrene, as seen with this patient's bullous lesions and crepitus. Additionally, the lecithinaseactivity of this toxin causes hemolysis, which characteristically manifests as a double zone of hemolysison blood agar

A 17-year-old male comes to the physician because of painful genital sores, malaise, and fever for 3 days. He is sexually active with 3 female partners and does not use condoms consistently. His temperature is 38.3°C (101°F). Physical examination shows tender lymphadenopathy in the left inguinal region and multiple, punched-out ulcers over the penile shaft and glans. Microscopic examination of a smear from the ulcer is most likely to show which of the following? a)Eosinophilic intranuclear inclusions b)Basophilic intracytoplasmic inclusions c)Eosinophilic intracytoplasmic inclusions d)Basophilic intranuclear inclusions e)Bipolar-staining intracytoplasmic inclusions

(This patient's presentation of tender inguinal lymphadenopathy, multiple punched-out ulcers, and systemic signs of infection (e.g., malaise, fever) suggest herpes genitalis, which is typically caused by herpes simplex virus 2 (HSV-2). - Eosinophilic intranuclear inclusions on a smear of an ulcer or opened vesicle describe the Cowdry A inclusion bodies seen in infections by herpesviruses such as HSV-1, HSV-2, VZV, and CMV. These intranuclear inclusions are formed during herpesvirus replication within the nucleus. Another microscopic characteristic of herpesvirus infection is the formation of multinucleated giant cells as a result of the fusion of multiple infected cells (seen on Tzanck smear). HSV-2 can remain dormant in the sacral ganglia after the primary infection to cause recurrent genital herpes.

A 59-year-old man comes to the emergency department because of excruciating left knee pain for 4 days. He underwent a total knee arthroplasty of his left knee joint 4 months ago. He has hypertension and osteoarthritis. Current medications include glucosamine, amlodipine, and meloxicam. His temperature is 38.1°C (100.6°F), pulse is 97/min, and blood pressure is 118/71 mm Hg. Examination shows a tender, swollen left knee joint; range of motion is limited by pain. Analysis of the synovial fluid confirms septic arthritis, and the prosthesis is removed. Which of the following is the most likely causal organism? a)Staphylococcus epidermidis b)Escherichia coli c)Staphylococcus aureus d)

(This patient's presentation with a tender and swollen knee and low-grade fever is consistent with septic arthritis. His history of arthroplasty makes a bacterial prosthetic joint infection most likely, even 4 months after the operation.) - Staphylococcus epidermidis has the ability to form a biofilm. It can therefore grow on the surface of foreign material such as prosthetic joints, prosthetic cardiac valves, IV catheters, and pacemakers. The patient underwent an arthroplasty in his left knee 4 months ago and is now presenting with septic arthritis, which makes a foreign body infection very likely. S. epidermidis is the most common cause of prosthetic infections 3-12 months after surgery. If the patient's infection had developed either < 3 months or > 12 months after surgery, Staphylococcus aureus would be the most likely causal organism.

A 57-year-old man comes to the physician because of generalized malaise, yellowish discoloration of the eyes, and pruritus on the back of his hands that worsens when exposed to sunlight for the past several months. He has not seen a physician in 15 years. Physical examination shows scleral icterus and mild jaundice. There is a purpuric rash with several small vesicles and hyperpigmented lesions on the dorsum of both hands. The causal pathogen of this patient's underlying condition was most likely acquired in which of the following ways? a)Bathing in freshwater b)ingestion of raw shellfish c)Needlestick injury d)Inhalation of spores e)Sexual contact

(This patient's scleral icterus and jaundice should raise suspicion for liver disease. His pruritic rash that worsens upon exposure to sunlight is suggestive of porphyria cutanea tarda, a condition that is associated with hepatitis C virus infection) -Needlestick injury: Needlestick injury is a likely cause of this patient's hepatitis C infection as it is mainly transmitted parenterally. Therefore, certain populations such as health care workers and intravenous drug users are at increased risk of contracting the virus. Patients who received blood transfusions before 1992 are also at increased risk for hepatitis C, as donor blood was not screened for the virus before 1992. Other routes of transmission include vertical (from mother to fetus) and, rarely, sexual

A 23-year-old man comes to the physician because of a 2-day history of profuse watery diarrhea and abdominal cramps. Four days ago, he returned from a backpacking trip across Southeast Asia. Physical examination shows dry mucous membranes and decreased skin turgor. Stool culture shows gram-negative, oxidase-positive, curved rods that have a single polar flagellum. The pathogen responsible for this patient's condition most likely has which of the following characteristics? a)Requires low infectious dose to establish infection b)Acts by activation of guanylate cyclase C) Grows well in medium with pH of 9 d)Forms spores in unfavorable environment

(This patient's stool microscopy findings correspond with the morphological appearance of Vibrio cholerae) - Grows well in medium with pH of 9: Vibrio cholerae is a gram-negative, oxidase-positive, curved rod. It produces an enterotoxin, which permanently activates a GS protein and thereby results in increased intracellular cAMP levels. This leads to secretion of chloride ions and water in the intestines, resulting in the typical rice-water diarrhea. V. cholerae is acid-labile, growing well in an alkaline medium (e.g. with a pH of 9). Accordingly, stomachacidity provides a natural barrier against V. cholera, so that a high amount of pathogens are required to cause infection (high infectious dose). A decrease in gastric acidity (e.g., due to treatment with proton pump inhibitors) reduces the infectious dose significantly.

A 30-year-old woman comes to the emergency department because of fever, watery diarrhea, and abdominal cramping for the past 24 hours. She recently went to an international food fair. Her temperature is 39°C (102.2°F). Physical examination shows increased bowel sounds. Stool cultures grow gram-positive, spore-forming, anaerobic rods that produce alpha toxin. The responsible organism also causes which of the following physical examination findings? a)Diffuse, flaccid bullae b)Facial paralysis c)Subcutaneous crepitus d)Petechial rash

(This patient's symptoms and stool culture are consistent with Clostridium perfringens enterocolitis.) - subcutaneous crepitus: Clostridium perfringens, the organism responsible for this patient's symptoms, can also cause gas gangrene, which classically presents with subcutaneous emphysema and palpable crepitus. The spores of these obligate anaerobic rods survive in undercooked food; when ingested, bacteria release heat-labile enterotoxins that cause food poisoning with watery diarrhea and severe abdominal cramping. This condition is rarely complicated by a syndrome that resembles necrotizing enteritis.

A 28-year-old woman with a history of intravenous drug use is brought to the emergency department because of a 1-day history of fatigue, yellow eyes, confusion, and blood in her stools. She appears ill. Her temperature is 38.1°C (100.6°F). Physical examination shows pain in the right upper quadrant, diffuse jaundice with scleral icterus, and bright red blood in the rectal vault. Further evaluation demonstrates virions in her blood, some of which have a partially double-stranded DNA genome while others have a single-stranded RNA genome. They are found to share an identical lipoprotein envelope. This patient is most likely infected with which of the following pathogens? a)Hepevirus b)Deltavirus c)Flavivirus d)Picornavirus

(This presents with features of acute viral hepatitis (e.g., fever, jaundice, right upper quadrant tenderness) and hepatic failure (e.g., confusion due to encephalopathy, rectal bleeding due to coagulopathy). Her history of intravenous drug use puts her at risk of infection with hepatitis B virus (HBV), which is an enveloped partially double-stranded DNA virus. The presence of virions with a single-stranded RNA genome and an envelope with identical lipoproteins as the HBV virus indicates that the patient is concurrently infected with the hepatitis D virus.) - deltavirus: Hepatitis B virus (HBV) and hepatitis D virus (HDV) are transmitted sexually, parenterally (e.g., contaminated shared needles), or perinatally. Acute HBV infection is usually mild or asymptomatic and resolves within a few weeks or months. However, 5% of adult patients will develop chronic HBV infection. HDV, on the other hand, is a defective virus that is dependent on the HBsAg coat of HBV for entry into hepatocytes. Therefore, HDV can only cause infection if simultaneous infection with HBVoccurs or if the patient already has a chronic HBV infection. Simultaneous infection of HBV with HDVusually has a protracted course but superinfection by HDV in a patient with pre-existing HBV infectionoften has a rapid course characterized by fulminant hepatic failure, as seen here.

A 27-year-old man comes to the physician with throbbing right scrotal pain for 1 day. He has also had a burning sensation on urination during the last 4 days. He is sexually active with multiple female partners and does not use condoms. Physical examination shows a tender, palpable swelling on the upper pole of the right testicle; lifting the testicle relieves the pain. A Gram stain of urethral secretions shows numerous polymorphonuclear leukocytes but no organisms. Which of the following is the most likely causal pathogen of this patient's symptoms? a)Neisseria gonorrhoeae b)Pseudomonas aeruginosa c)Mycobacterium tuberculosis d)Treponema pallidum e)Chlamydia trachomatis

(This sexually active young man has dysuria, a tender swelling on the right testicle, and a positive Prehn sign (lifting the testicle relieves the pain), indicating epididymitis.) - In a sexually active postpubertal man < 35 years, the most common causes of infectious epididymitisare Chlamydia trachomatis and Neisseria gonorrhoeae. If the Gram stain shows no organisms (especially no diplococci, which would be expected in gonorrhea), C. trachomatis should be suspected, as its cell wall does not stain. It can be diagnosed by nucleic acid amplification. Treatment of C. trachomatisepididymitis involves ceftriaxone and doxycycline. Other causes of infectious epididymitis in young boys who are not sexually active would be Mycoplasma and enteroviruses.

A 47-year-old woman comes to the physician because of a 3-day history of fever, fatigue, loss of appetite, cough, and chest pain. Physical examination shows diffuse inspiratory crackles over the left lung field. An x-ray of the chest shows hilar lymphadenopathy and well-defined nodules with central calcifications. Urine studies show the presence of a polysaccharide antigen. A biopsy specimen of the lung shows cells with basophilic, crescent-shaped nuclei and pericellular halos located within macrophages. This patient's history is most likely to show which of the following? a)Treatment with inhaled glucocorticoids b)Visit to Arizona desert c)Recent trip to Brazil d)Hobby of sport fishing e)Previous mycobacterial infection f)Exposure to bat droppings

(This woman has pneumonia (fever, fatigue, loss of appetite, cough, chest pain, crackles), hilar lymphadenopathy, a polysaccharide antigen in the urine, and a biopsy specimen of the lung that show cells yeast cells within macrophages, all of which are features of Histoplasma capsulatum infection.

A 47-year-old woman comes to the physician because of a 3-day history of fever, fatigue, loss of appetite, cough, and chest pain. Physical examination shows diffuse inspiratory crackles over the left lung field. An x-ray of the chest shows hilar lymphadenopathy and well-defined nodules with central calcifications. Urine studies show the presence of a polysaccharide antigen. A biopsy specimen of the lung shows cells with basophilic, crescent-shaped nuclei and pericellular halos located within macrophages. This patient's history is most likely to show which of the following? a)Treatment with inhaled glucocorticoids b)Visit to Arizona desert c)Recent trip to Brazil d)Hobby of sport fishing e)Previous mycobacterial infection f)Exposure to bat droppings

(This woman has pneumonia (fever, fatigue, loss of appetite, cough, chest pain, crackles), hilar lymphadenopathy, a polysaccharide antigen in the urine, and a biopsy specimen of the lung that show cells yeast cells within macrophages, all of which are features of Histoplasma capsulatum infection.) - Exposure to bat droppings: Bat and bird droppings can carry Histoplasma capsulatum, particularly in endemic areas (e.g., the Ohio and Mississippi river valleys). Inhalation of the fungus causes histoplasmosis, a condition that manifests with primary pulmonary disease or disseminated disease, especially in patients who are immunocompromised.

A 39-year-old woman is brought to the emergency department because of fevers, chills, and left lower quadrant pain. Her temperature is 39.1°C (102.3°F), pulse is 126/min, respirations are 28/min, and blood pressure is 80/50 mm Hg. There is blood oozing around the site of a peripheral intravenous line. Pelvic examination shows mucopurulent discharge from the cervical os and left adnexal tenderness. Laboratory studies show: platelet 14,200 fibrinogen 83 low d dimer 965 high When phenol is applied to a sample of the patient's blood at 90°C, a phosphorylated N-acetylglucosamine dimer with 6 fatty acids attached to a polysaccharide side chain is identified. A blood culture is most likely to show which of the following? a)Coagulase-positive, gram-positive cocci forming mauve-colored colonies on methicillin-containing agar b)Encapsulated, gram-negative coccobacilli forming grey-colored colonies on charcoal blood agar c)Spore-forming, gram-positive bacilli forming yellow colonies on casein agar d)Lactose-fermenting, gram-negative rods forming pink colonies on MacConkey agar

(This woman's fever, tachycardia, and hypotension suggest septic shock, and her laboratory studies (low fibrinogen, elevated D-dimer, and thrombocytopenia) indicate that she has also developed disseminated intravascular coagulation (DIC). The identification of a phosphorylated N-acetylglucosamine dimer with fatty acids attached to a polysaccharide side chain indicates the presence of an organism with a lipopolysaccharide (LPS) endotoxin.) - Escherichia coli is a lactose-fermenting, gram-negative rod that grows as pink colonies on MacConkey agar. Like all gram-negative organisms, its outer cellular membrane is composed of lipopolysaccharide, which is also known as endotoxin. The lipid A component of endotoxin activates tissue factor, the complement system, and macrophages, resulting in fever, hypotension, and DIC. In light of this patient's left lower quadrant pain with mucopurulent discharge from the cervix, a tubo-ovarian abscess (TOA) is the most likely source of the E. coli bacteremia.

A 29-year-old nulligravid woman comes to the physician for evaluation of infertility. She has been unable to conceive for 14 months. One year ago, she stopped taking the oral contraceptive pill, which she had been taking since she was 17. Her husband's semen analysis was normal. Four years ago, she had an episode of a pelvic tenderness and vaginal discharge that resolved without treatment. Menses occur at regular 28-day intervals. Before her marriage, she was sexually active with 5 male partners and used condoms inconsistently. She is 169 cm(5 ft 6 in) tall and weighs 86 kg (190 lb); BMI is 31.6 kg/m2. Physical examination shows no abnormalities. Which of the following is the most likely cause of this patient's infertility? a)Insulin resistance b)Loss of ciliary action c)Ectopic endometrial tissue d)Primary ovarian insufficiency

(This woman's previously engaged in high-risk sexual behavior. This put her at risk for sexually transmitted infections and pelvic inflammatory disease (PID), which was the likely cause of her episode of pelvic tenderness and vaginal discharge.) - Loss of ciliary action: Infertility that occurs following PID is a result of tubal scarring, which results in loss of ciliary action and tubal occlusion and/or a tubo-ovarian abscess. The risk of infertility increases with the number of episodes of PID. Besides infertility, other complications associated with PID include tubo-ovarian abscess, pelvic peritonitis, sepsis, Fitz-Hugh-Curtis syndrome, and ectopic pregnancy. Symptomatic PIDtypically presents with bilateral lower abdominal pain or tenderness, as well as evidence of inflammationof the genital tract, but symptoms vary and may be very subtle or absent. Empirical antibiotic treatmentof PID (with ceftriaxone and either doxycycline or azithromycin) is directed against the two most common causal pathogens: Chlamydia trachomatis and Neisseria gonorrhoeae.

A 39-year-old woman comes to the physician because of a 5-day history of pain and stiffness in her hands and wrists and a nonpruritic generalized rash. The stiffness is worst in the morning and improves after 15-20 minutesof activity. She had fever and a runny nose 10 days ago that resolved without treatment. She is sexually active with a male partner and uses condoms inconsistently. She works as an elementary school teacher. Her temperature is 37.3°C (99.1°F), pulse is 78/min, and blood pressure is 120/70 mm Hg. Examination shows swelling, tenderness, and decreased range of motion of the wrists as well as the metacarpophalangeal and proximal interphalangeal joints. There is a lacy macular rash over the trunk and extremities. Laboratory studies, including erythrocyte sedimentation rate and anti-nuclear antibody and anti-dsDNA serology, show no abnormalities. Which of the following is the most likely cause of this patient's symptoms? a)Systemic lupus erythematosus b)Psoriatic arthritis c)Parvovirus arthritis d)Reactive arthritis e)

(This young woman with a recent history of flu-like symptoms, a nonpruritic generalized rash, and acute bilateral upper extremity arthralgias likely contracted a self-limited disease given her exposure to young children.) - Parvovirus B19 infection, also known as erythema infectiosum or fifth disease, is a common childhood disease that can also affect adults. The disease typically manifests with mild febrile illness, occasionally followed by a maculopapular rash beginning on the perioral areas of the face several days later and spreading to the extremities from there. As the rash clears, it adopts a lacy and reticular pattern, as seen in this patient. More commonly in adults, parvovirus B19 infection may also present with symptoms of symmetric polyarthritis in the fingers, hands, knees, and ankles while the rash can be absent. The disease is usually self-limited and treatment is supportive, involving analgesics and NSAIDs. A short course of prednisone can be prescribed for parvovirus B19-associated arthritis.

An investigator is developing a new vaccine. After injecting the agent, the immune response is recorded by measuring vaccine-specific antibodies at subsequent timed intervals. To induce the maximum immunogenic response, this vaccine should have which of the following properties? a)Autologous denatured protein b)Foreign intact polysaccharide bound to protein c)Chemically inactivated microorganism d)Foreign intact polysaccharide e)Foreign denaturated protein f)Weakened live microorganisms

(To induce the maximum immunogenic response, a vaccine should activate both the humoral and cellular immune system.) - Live attenuated vaccines are derived from weakened live microorganisms. These types of vaccines have lost their pathogenicity but retain the ability to activate the cellular and humoral immune responses. They confer strong and often lifelong immunity but are contraindicated in patients with weakened immune systems (e.g., AIDS, pregnancy). Examples of this type of vaccine include the MMR vaccine and the varicella vaccine.

An 81-year-old man comes to the emergency department with severe left ear pain and drainage for 3 days. He has a history of poorly-controlled type 2 diabetes mellitus. He appears uncomfortable. Physical examination of the ear shows marked periauricular erythema, exquisite tenderness on palpation, and granulation tissue in the external auditory canal. The most likely causal pathogen produces an exotoxin that acts by a mechanism most similar to a toxin produced by which of the following organisms? a)Corynebacterium diphtheriae b)Bordetella pertussis c)Shigella dysenteriae d)Staphylococcus aureus

(Unilateral severe ear pain and drainage with granulation tissue in the external auditory canal in a patient with poorly controlled diabetes mellitus are highly suggestive of malignant otitis externa caused by Pseudomonas aeruginosa. This organism produces exotoxin A, which induces host cell death via inhibition of elongation factor 2.) - Corynebacterium diphtheriae produces diphtheria toxin, which inhibits protein synthesis and induces cell death by the same mechanism as Pseudomonas exotoxin A, via inactivation of elongation factor 2. C. diphtheriae causes diphtheria. Other virulence factors produced by P. aeruginosa include phospholipase C, endotoxins, and pigments (pyoverdin and pyocyanin).

A 28-year-old primigravid woman at 38 weeks' gestation is brought to the emergency department in active labor. She has not had regular prenatal care. She has a history of HIV infection but is not currently on antiretroviral therapy. Her previous viral load is unknown. Treatment with intravenous zidovudine is begun to reduce perinatal transmission of the virus. Which of the following processes is most likely affected by this drug? a)Integration of viral genome b)Action of viral RNA polymerase c)Cleavage of viral polypeptides d)Elongation of viral DNA

(Zidovudine is a nucleoside analog that lacks a hydroxyl group at the 3' end.) - Elongation of viral DNA: Zidovudine is a nucleoside reverse transcriptase inhibitor that is structurally similar to thymidine. However, instead of a hydroxyl group at the 3' end, it has an azido group that prevents the formation of 3'→5' phosphodiester linkages needed for DNA replication. Thus, after incorporation into the viral genome as a thymidine analog, it prevents the elongation of viral DNA by HIV viral RNA-dependent DNA polymerase. Zidovudine is the drug of choice for HIV-positive women in labor; there is no evidence that it has a teratogenic effect in humans.

A 51-year-old man comes to the physician for follow-up evaluation. Nine months ago, he was diagnosed with acute viral hepatitis B infection. Physical examination shows no abnormalities. Serum studies show increased hepatic transaminase activity and a hepatitis B viral DNA load of 4286 IU/mL. Which of the following sets of findings is most likely in this patient? HbeAg + anti Hbc IgG +

Up to 10% of patients who develop acute hepatitis B infection will develop chronic hepatitis B infection, which is characterized by the persistence of HBsAg and HBV DNA for more than 6 months after the initial infection, and the absence of anti-HBs. IgG anti-HBc may be present or absent with chronic hepatitis B infection. Chronic hepatitis B infection can be classified as "active" (marked viral replication characterized by increased HBeAg) or "inactive" (low or no viral replication characterized by an absence of serum HBeAg). Patients with active chronic hepatitis B would have an HBV DNA load > 2000 IU/L and/or increased hepatic transaminase levels.


Kaugnay na mga set ng pag-aaral

direct and digital marketing midterm :(

View Set

Chapter 10: Monopolistic Competition and Oligopoly

View Set

Scientific Method, Theory, Hypothesis

View Set

EAQ Ch 58 Care of Patients with Liver Problems

View Set

Introduction to Communication Concepts

View Set

STA15_03_Lesson9_10_U32_Place an order_Check an order

View Set

logic & critical thinking HW 3a-e

View Set

Health Science Ethics - GMS 6871-005 - Chapter 7 - Liability and Health Care: Legal Aspects of Health Care

View Set

CHAPTER 16: UNDERSTANDING MONEY AND THE ROLE OF BANKING (721E85-2)

View Set

LEGAL, ETHICAL & MORAL ISSUES OF HEALTH CARE

View Set

Chapter 25 Pregnancy- Related Complications

View Set

World Geography A: Unit 5 - Exam

View Set

Retirement Other Insurance Concepts

View Set

Chapter 1 : Understanding the Manager's Job

View Set

Review Questions for Chapter 2 - The Chemical Foundation of Life

View Set